Универсальный внешний накопитель для всех iOS-устройств, совместим с PC/Mac, Android
Header Banner
8 800 100 5771 | +7 495 540 4266
c 9:00 до 24:00 пн-пт | c 10:00 до 18:00 сб
0 Comments

Содержание

Индуцированная электродвижущая сила и магнитный поток

Физика > Индуцированная ЭДС и магнитный поток

 

Рассмотрите электродвижущую силу, магнитный поток и закон электромагнитной индукции Фарадея: изменение магнитного потока в магнитном поле, гальванометр.

Закон индукции Фарадея: электродвижущая сила индуцируется через изменение в магнитном потоке.

Задача обучения

  • Разобраться в связи магнитного поля и электродвижущей силы.

Основные пункты

  • Изменение потока магнитного поля вызывает ЭДС.
  • Магнитный поток сквозь поверхность выступает составляющей магнитного поля.
  • Общая формула магнитного потока: ΦB = ∬AB ⋅ dA. Это интеграл всего магнитного поля, проходящего сквозь небольшие участки dА.

Термины

  • Гальванометр – аналоговый прибор измерения тока (G), который основывается на отклонении иглы, созданной силой магнитного поля.
  • Область вектора – вектор, чья величина расположена в конкретной области, а направление выступает перпендикулярным поверхностной площади.

Индуцированная ЭДС

Гальванометр использовал Фарадей, чтобы продемонстрировать способность магнитных полей создавать токи. Если переключатель закрыт, то магнитное поле скапливается в катушке на верхушке железного кольца и транспортируется на нижнее кольцо. Гальванометр применяют для выявления тока, индуцированного в отдельной катушке на дне.

Аппарат Фарадея показывает умение магнитного поля создавать ток. Перемены из-за верхней катушки индуцируют ЭДС, а также и ток в нижней катушке. При открытии/закрытии переключателя гальванометр регистрирует противоположное направление токов. Если контакт выключен или открыт, то ток не проходит сквозь аппарат

В экспериментах ученый заметил, что при закрытом выключателе гальванометр находит ток, перемещающийся в одном направлении на дне катушки. А вот при открытом, ток меняет направление.

Если же оставить выключатель в одном положении на долгое время (открытый или закрытый), то ток прекращает перемещаться. Перемена в магнитном поле формирует ток.

Главной также остается электродвижущая сила (ЭДС), которая к нему и приводит. Ток – результат ЭДС, созданной переменой магнитного поля. Причем на нее не влияет присутствие траектории прохождения тока.

Магнитный поток

Магнитный поток – составляющая магнитного поля. Выступает пропорциональным количеству линий поля, пронизывающих поверхность. Высчитывается по формуле:

ΦB = B ⋅ A = BAcosθ (В – величина магнитного поля, А – площадь поверхности, а θ – угол между линиями магнитного поля и перпендикуляром).

Давайте взглянем на магнитный поток для бесконечно малого элемента участка dA, где будем считать поле постоянным:

Все точки на поверхности обладают единым направлением – поверхностная нормаль. Магнитный поток сквозь точку выступает составляющей магнитного поля вдоль нормального направления

B = B ⋅ dA

Теперь общую поверхность А можно разделить на бесконечно малые элементы, а полный магнитный поток выступит интегралом поверхности:

ΦB = ∬A B ⋅ dA.


От чего зависит величина индукционного тока: получение переменной ЭДС

Принцип получения переменной ЭДС

Пусть в однородном магнитном поле постоянного магнита равномерно вращается рамка (рис. 2.1), активные стороны которой перпендикулярны чертежу и пересекающие линии магнитной индукции с некоторой линейной скоростью v по часовой стрелке. При этом в сторонах а и в рамки наводится ЭДС противоположной полярности. При пересечении линии между точками А и В в сторонах рамки полярность меняется на противоположную.

Рис. 2.1

Время Т одного полного изменения ЭДС (это время одного оборота рамки) называют периодом ЭДС. Изменение ЭДС со временем может быть изображено на временной диаграмме (рис. 2.2). Магнитный поток Ф через рамку будет равен:

Ф = ВS cos б, 2. 1

где В — вектор магнитной индукции;

S — площадь активной части рамки;

сos б — угол между нормалью к рамке n и вектором магнитной индукции.

Для исчерпывающего определения синусоиды достаточно указать ее амплитуду, период и начальную фазу. Рекомендуем читателю самостоятельно построить две-три синусоиды с разными начальными фазами.

Рис. 2.2

Необходимость определения начальной фазы. Необходимость определения начальной фазы вытекает из следующего простого примера.

Представьте себе два последовательно включенных генератора, частоты и амплитуды ЭДС которых одинаковы. Спрашивается: можно ли заранее определить, суммарное напряжение в каждый момент времени? Очевидно, что нельзя.

Согласное включение двух генераторов. Если генераторы имеют одинаковую начальную фазу, то кривые напряжения (синусоиды в данном случае) для каждого генератора, изображенные на одном и том же чертеже и в одинаковом масштабе, совпадут. Следовательно, общее (суммарное) напряжение обоих генераторов будет всегда вдвое больше напряжения каждого генератора в отдельности. Обычно такое включение генераторов называют согласным.

Встречное включение двух генераторов. Предположим, что один генератор имеет начальную фазу, равную нулю, а другой — равную 180°, т. е. величина напряжения первого генератора в любой момент времени имеет то же значение, что и величина напряжения второго, однако знаки напряжений («+» или «-«) не будут совпадать.

В момент времени, когда напряжение первого генератора положительно, напряжение второго генератора отрицательно, и наоборот.

Учитывая, что напряжения складываются алгебраически, приходим к выводу, что результирующее напряжение в каждый момент времени равно нулю. Заметим, что если фазы генераторов отличаются на 180°, то иногда говорят, что генераторы работают в противофазе или соединены встречно.

Сдвиг фазы. Очевидно, что две синусоиды, имеющие разные начальные фазы, как бы сдвинуты одна относительно другой по горизонтали. Поэтому разность начальных фаз двух синусоид и называют обычно сдвигом фазы.

На рис. 2.3 показано простейшее устройство для получения переменного тока. По катушке проходит постоянный ток, и, следовательно, магнитное поле также постоянно. Стальной сердечник придает магнитным линиям желательную форму: между полюсами получается приблизительно однородное поле. В этом поле равномерно вращается прямоугольная рамка. Концы рамки соединены при помощи скользящих контактов с вольтметром.

Как уже сказано, магнитный поток, созданный катушкой, является постоянным. Но та его доля, которая сцеплена с вращающейся рамкой, будет неодинакова в разные моменты времени.

Изменение величины магнитного потока, пронизывающего виток, происходит непрерывно, хотя поток, создаваемый электромагнитом, остается неизменным. Следовательно, в рамке будет наводиться ЭДС. И действительно, опыт показывает, что стрелка вольтметра отклоняется.

Рис. 2.3 Получение переменного тока

Стрелка вольтметра попеременно отклоняется вправо и влево от нулевого положения.

Поскольку при вращении рамки пересекающий ее магнитный поток все время меняется, то по закону электромагнитной индукции в ней будет наводиться ЭДС индукции. Если период измеряется в секундах, то частота измеряется в герцах. В большинстве стран, включая Россию, промышленная частота переменного тока составляет 50 Гц (в США и Японии — 60 Гц). Величина промышленной частоты переменного тока обусловлена технико-экономическими соображениями. Если она слишком низка, то увеличиваются габариты электрических машин и, следовательно, расход материалов на их изготовление; заметным становится мигание света в электрических лампочках. При слишком высоких частотах увеличиваются потери энергии в сердечниках электрических машин и трансформаторах. Поэтому наиболее оптимальными оказались частоты 50 — 60 Гц. Однако в некоторых случаях используются переменные токи как с более высокой, так и с более низкой частотой. Например, в самолетах применяется частота 400 Гц. На этой частоте можно значительно уменьшить габариты и вес трансформаторов и электромоторов, что для авиации более существенно, чем увеличение потерь в сердечниках.

Катушка индуктивности является пассивным компонентом электронных схем, основное предназначение которой является сохранение энергии в виде магнитного поля. Свойство катушки индуктивности чем-то схоже с конденсатором, который хранит энергию в виде электрического поля.

Индуктивность (измеряется в Генри) — это эффект возникновения магнитного поля вокруг проводника с током. Ток, протекающий через катушку индуктивности, создает магнитное поле, которое имеет связь с электродвижущей силой (ЭДС) оказывающее противодействие приложенному напряжению.

Возникающая противодействующая сила (ЭДС) противостоит изменению переменного напряжения и силе тока в катушке индуктивности. Это свойство индуктивной катушки называется индуктивным сопротивлением. Следует отметить, что индуктивное сопротивление находится в противофазе к емкостному реактивному сопротивлению конденсатора в цепи переменного тока. Путем увеличения числа витков можно повысить индуктивность самой катушки.

Индуктивность в электрических цепях

В то время как конденсатор оказывает сопротивление изменению переменного напряжения, индуктивность же сопротивляется переменному тока. Идеальная индуктивность не будет оказывать сопротивление постоянному току, однако, в реальности все индуктивные катушки сами по себе обладают определенным сопротивлением.

В целом, отношение между изменяющимися во времени напряжением V(t) проходящим через катушку с индуктивностью L и изменяющимся во времени током I(t), проходящим через нее можно представить в виде дифференциального уравнения следующего вида:

Когда переменный синусоидальной ток (АС) протекает через катушку индуктивности, возникает синусоидальное переменное напряжение (ЭДС). Амплитуда ЭДС зависит от амплитуды тока и частоте синусоиды, которую можно выразить следующим уравнением:

где ω является угловой частотой резонансной частоты F:

Причем, фаза тока отстает от напряжения на 90 градусов. В конденсаторе же все наоборот, там ток опережает напряжение на 90 градусов. Когда индуктивная катушка соединена с конденсатором (последовательно либо параллельно), то образуется LC цепь, работающая на определенной резонансной частоте.

Индуктивное сопротивление ХL определяется по формуле:

где ХL — индуктивное сопротивление, ω — угловая частота, F — частота в герцах, и L индуктивность в генри.

Индуктивное сопротивление — это положительная составляющая импеданса. Оно измеряется в омах. Импеданс катушки индуктивности (индуктивное сопротивление) вычисляется по формуле:

Применение катушек индуктивности

Индуктивности широко используются в аналоговых схемах и схемах обработки сигналов. Они в сочетании с конденсаторами и другими радиокомпонентами образуют специальные схемы, которые могут усилить или отфильтровать сигналы определенной частоты.

Катушки индуктивности получили широкое применение начиная от больших катушек индуктивности, таких как дроссели в источниках питания, которые в сочетании с конденсаторами фильтра устраняют остаточные помехи и другие колебания на выходе источника питания, и до столь малых индуктивностей, которые располагаются внутри интегральных микросхем.

Две (или более) катушки индуктивности, которые соединены единым магнитным потоком, образуют трансформатор, являющимся основным компонентом схем работающих с электрической сетью электроснабжения. Эффективность трансформатора возрастает с увеличением частоты напряжения.

По этой причине, в самолетах используется переменное напряжение с частотой 400 герц вместо обычных 50 или 60 герц, что в свою очередь позволяет значительно сэкономить на массе используемых трансформаторов в электроснабжении самолета.

Так же индуктивности используются в качестве устройства для хранения энергии в импульсных стабилизаторах напряжения, в высоковольтных электрических системах передачи электроэнергии для преднамеренного снижения системного напряжения или ограничения ток короткого замыкания.

Законы Фарадея и Ленца

Электрические токи создают магнитные эффекты. А возможно ли, чтобы магнитное поле порождало электрическое? Фарадей обнаружил, что искомые эффекты возникают вследствие изменения МП во времени.

Когда проводник пересекается переменным магнитным потоком, в нем индуцируется электродвижущая сила, вызывающая электроток. Системой, которая генерирует ток, может быть постоянный магнит или электромагнит.

Явление электромагнитной индукции регулируется двумя законами: Фарадея и Ленца.

Закон Ленца позволяет охарактеризовать электродвижущую силу относительно ее направленности.

Важно! Направление индуцированной ЭДС такое, что вызванный ею ток стремится противостоять создающей его причине.

Фарадей заметил, что интенсивность индуцированного тока растет, когда быстрее изменяется число силовых линий, пересекающих контур. Другими словами, ЭДС электромагнитной индукции находится в прямой зависимости от скорости движущегося магнитного потока.

ЭДС индукции

Формула ЭДС индукции определена как:

Е = — dФ/dt.

Знак «-» показывает, как полярность индуцированной ЭДС связана со знаком потока и меняющейся скоростью.

Получена общая формулировка закона электромагнитной индукции, из которой можно вывести выражения для частных случаев.

Вращающаяся катушка

Работа генератора электроэнергии основана на вращении контура в МП, имеющего N витков.

ЭДС индуцируется в электроцепи всегда, когда магнитный поток ее пересекает, в соответствии с определением магнитного потока Ф = B x S х cos α (магнитная индукция, умноженная на поверхностную площадь, через которую проходит МП, и косинус угла, образованного вектором В и перпендикулярной линией к плоскости S).

Из формулы следует, что Ф подвержен изменениям в следующих случаях:

  • меняется интенсивность МП – вектор В;
  • варьируется площадь, ограниченная контуром;
  • изменяется ориентация между ними, заданная углом.

В первых опытах Фарадея индуцированные токи были получены путем изменения магнитного поля В. Однако можно индуцировать ЭДС, не двигая магнит или не меняя ток, а просто вращая катушку вокруг своей оси в МП. В данном случае магнитный поток меняется из-за изменения угла α. Катушка при вращении пересекает линии МП, возникает ЭДС.

Если катушка вращается равномерно, это периодическое изменение приводит к периодическому изменению магнитного потока. Или количество силовых линий МП, пересекаемых каждую секунду, принимает равные значения с равными интервалами времени.

Вращение контура в МП

Важно! Наведенная ЭДС меняется вместе с ориентацией с течением времени от положительной до отрицательной и наоборот. Графическое представление ЭДС представляет собой синусоидальную линию.

Для формулы ЭДС электромагнитной индукции применяется выражение:

Е = В х ω х S x N x sin ωt, где:

  • S – площадь, ограниченная одним витком или рамкой;
  • N – количество витков;
  • ω – угловая скорость, с которой вращается катушка;
  • В – индукция МП;
  • угол α = ωt.

На практике в генераторах переменного тока часто катушка остается неподвижной (статор), а электромагнит вращается вокруг нее (ротор).

Взаимоиндукция

Если две катушки расположены рядом, то в них наводится ЭДС взаимоиндукции, зависящая от геометрии обеих схем и их ориентации относительно друг друга. Когда разделение цепей возрастает, взаимоиндуктивность снижается, так как уменьшается соединяющий их магнитный поток.

Взаимоиндукция

Пусть имеется две катушки. По проводу одной катушки, обладающей N1 витками, протекает ток I1, создающий МП, проходящее через катушку с N2 витками. Тогда:

  1. Взаимоиндуктивность второй катушки относительно первой:

М21 = (N2 x F21)/I1;

  1. Магнитный поток:

Ф21 = (М21/N2) x I1;

  1. Найдем индуцированную ЭДС:

Е2 = — N2 x dФ21/dt = — M21x dI1/dt;

  1. Идентично в первой катушке индуцируется ЭДС:

Е1 = — M12 x dI2/dt;

Важно! Электродвижущая сила, вызванная взаимоиндукцией в одной катушке, всегда пропорциональна изменению электротока в другой.

Взаимную индуктивность можно признать равной:

М12 = М21 = М.

Соответственно, E1 = — M x dI2/dt и E2 = M x dI1/dt.

М = К √ (L1 x L2),

где К – коэффициент связи между двумя индуктивностями.

Явление взаимоиндукции используется в трансформаторах – электроаппаратах, позволяющих изменить значение напряжения переменного электротока. Аппарат представляет собой две катушки, намотанные вокруг одного сердечника. Ток, присутствующий в первой, создает меняющееся МП в магнитопроводе и электроток в другой катушке. Если количество витковых оборотов первой обмотки меньше, чем другой, напряжение увеличивается, и наоборот.

Кроме генерирования, трансформации электроэнергии магнитная индукция применяется в иных устройствах. Например, в магнитных левитационных поездах, которые двигаются не в непосредственном контакте с рельсами, а на несколько сантиметров выше из-за электромагнитной силы отталкивания.

>Видео

Использование явления электромагнитной индукции в технике — МегаЛекции

Физическая суть явления электромагнитной индукции

Величина индуцированной ЭДС

Потокосцепления

Использование явления электромагнитной индукции в технике

Явление самоиндукции

Энергия магнитного поля

Вихревые токи

Литература

Данилов Н. А., Иванов Н.М., «Общая электротехника с основами электроники», М .: 2005.

Гаврилов В.А. «Общая электротехника с основами электроники», М .: 1980

 

 

Физическая суть явления электромагнитной индукции

Явлением елекипромагнитнои индукции называется виникнення ЭДС в проводнике при пересечении проводником магнитних силовых линий или при изменении величины магнитного потока, пронизывающего контур.

Изменение величины магнитного потока внутри катушки вызывает наведення ЭДС.

Для определения направления ЭДС, индуцированной в контуре или в катушке, что возникает при изменении магнитного потока в них используется правило Ленца:

Индуцированная ЕPC всегда направлена ​​так, что своим смрумом и магнитным потоком противодействует причине, что ее вызвало

Например, если постоянный магнит вносится в катушку то в ней возникает индукционный ток такого направления, противодействует движения магнита к катушке, то есть сверху катушки возникает одноименный полюс, отталкивает магнит Если магнит, который находился внутри катушки, отдаляется от нее, то направление индукционного тока будет таким, еще сверху катушки возникает противолежащий полюс, который притягивает магнит к катушке, то есть препятствует его удалению, что, в свою очередь, и было причиной возникающую тока

Величина индуцированной ЭДС

Величина индуцированной ЭДС при перемещении проводника в магнитном поле определяется по формуле

е=Blv sinα,

где е – индуцированная ЭДС, В; v- скорость движения проводника, м / с; l – длина проводника, м; α – угол между направлением вектора магнитной индукции и направлением движения проводника, градус.



При перемещении проводника под углом 90 ° к направлению магнитной индукции поля индуцированная ЭДС имеет максимальную величину, sin 90 ° = 1 при других углах движения величина ЭДС меньшая. Это объясняется тем, что при перемещении под углом 90 ○ проводник за единицу времени пересекает самую килькисть магнитных силовых линий.

 
 

Когда индуцированная ЭДС возникает в контуре или внутри катушки с счет магнитного потока, меняется, то величина ЭДС зависит от скорости изменения магнитного потоку и определяется по формуле

где e – индуцированная ЭДС, В; dФ = Ф1-Ф2 – прирост магнитного потока, Вб; dt = t1-t2 прирост времени: dФ / dt – скорость изменения магнитного потока, убьет / с.

Знак «минус» отображает физическую суть правила Ленца, то есть противодействие индуцированной ЭДС причине, что ее вызвало. Вообще электромагнитная индукция – это явление перетворення механической энергии в электрическую.

 

Потокосцепления

Если через катушку проходит ток, то образуется магнитный поток Ф, сцепленный с витками катушки N. Если все магнитные линии сцеплении со всеми витками катушки (см. Рис.), То потокосцепления выражается формулой

Ψ = NФ,

где Ψ – потокосцепления, Вб.

Если магнитный поток меняется, то изменение потокосцепления определяется так:

 

 
 

Ψ = NdФ,

Тогда индуцированной ЭДС можно выразить через потокосцепления

Закон электромагнитной индукции выражается так: электродвижущая сила, индуцируется в замкнутом контуре при изменении сцепленного с ним магнитного потока, равна скорости изменения потокосцепления.

Использование явления электромагнитной индукции в технике

Явление электромагнитной индукции широко применяется в технике, например на электростанциях в генераторах большой мощности, которые превращают механическую энергию и электрическую. Это явление применяют также в устройствах, работающих с достаточно малыми мощностями (звукознимачi электропроигрывателей, которые обеспечивают воспроизведение грампластинок; магнитофоны; электродинамические микрофоны) Мощности развивающихся в этих устройствах, измеряются долями микроватт.

Явление самоиндукции

Если с помощью потенциометра изменять напряжение, подаваемое на тороидальных катушку, то при изменении величины тока в обмотке катушки меняется магнитный поток, и поэтому в катушке индукуватиметься ЭДС, называется ЭДС самоиндукции.

Саяомдукциею называется явление возникновения в проводнике или в катушке ЭДС, которая образуется в результате изменения собственного тока и созданного им магнитного потока.

Явление самоиндукции – это частный случай электромагнитной индукции, оно наблюдается во всех электрических колах, где меняется величина тока. В цепях переменного тока ЭДС самоиндукции возникает непрерывно, а в цепях постоянного тока – только в трех случаях, а именно:

при замыкании цепи, поскольку ток в цепи возрастает от нуля до некоторой величины, определяется по закону Ома;

при размыкании цепи, поскольку ток уменьшается от существующей величины до нуля;

при изменении величины тока с помощью реостата или потенциометра.

Направление ЭДС самоиндукции определяется по правилу Ленца, то есть при увеличении величины тока в цепи возникает ЭДС самоиндукции, противоположная по направлению к току, так, что противодействует его росту, который является причиной возникновения ЭДС.

Если ток в цепи уменьшается, то ЭДС самоиндукции имеет тот же направление, что и ток, уменьшается, то есть препятствует его уменьшению. Таким образом, ЭДС противодействует уменьшению тока, поддерживая его.

 
 

коэффициент самоиндукции и индуктивность обозначается буквой Д и Единицей индуктивности является генри (Гн).

 
 

Формула ЭДС самоиндукции часто спрашивают так:

Отсюда величина ЭДС самоиндукции прямо пропорциональна скорости изменения тока и индуктивности катушки.

 
 

Рассмотрим физическую суть индуктивности L, определяется формулой

 

Из формулы L = фn / I следует, что индуктивностью катушки называется величина, характеризующая способность данного катушки создавать определенную величину магнитного потоку при токе в катушке 1 А.

Итак, чем больше индуктивность катушки, тем больше величина магнитного потока, созданного током, и тем больше ЭДС самоиндукции при данной скорости изменения тока.

Если в формуле L = фn / I взять единицы величин N, Ф, И, может L = 1 генри. Отсюда единица индуктивности один генри – это индуктивность такого контура, с которым сцеплен магнитний поток один вебер при токе в контуре в один ампер.

Индуктивность катушки равна одному генри, если в ней при изменении тока со скоростью один ампер за одну секунду индуцируется ЭДС самоиндукции в один вольт.

В электронных схемах часто используют катушки с индуктивностью, значительно меньше одного генри, а именно, моли и микрогенри 1 Гн = 103 мГн; 1 Гн = 106 мкГн. В формуле Ldi / dt можно заменить Ldi на dΨ. Тогда ЭДС самоиндукции определится так, как и для электромагнитной индукции:

 
 

 

На практике иногда необходимы безиндукцийни катушки, в которых L = 0 и при изменении тока в которых не возникает ЭДС самоиндукции. Чтобы получить такие катушки, их обмотки выполняют бифилярной, то есть когда в соседних витках катушки ток имеет противоположные направления и магнитные потоки созданные этими витками, имеют противоположные направления. Тогда общий магнитный поток катушки равен нулю. Следовательно, при изменении тока в витках катушки ЭДС самоиндукции не возникает.

Чтобы получить катушки с переменной индуктивностью, в таких котущках индуктивность меняют следующими способами:

1) меняют магнитную проницаемость посредством внесения в катушку или вынесением из нее различных сердечников

2) изменяют количество витков катушки с помощью переключателя.

3) применяют устройство, называемое вариометром.

Энергия магнитного поля

Если через проводник или катушку проходит ток, то часть электроэнергии расходуется на преодоление сопротивления проводника и превращается в тепло, а часть образует магнитное поле, в котором накапливается некоторая часть энергии превращается в потенциальну энергию.

При размыкании цепи ток уменьшается до нуля в течение очень малого промежутка времени, а также уменьшается до нуля и магнитное поле. При этом в проводнике или катушке индуцируется ЭДС самоиндукция за счет накопленной в магнитном поле энергии.

 
 

Итак, энергия магнитного поля

 

Пример

В магнитном поле катушки с индуктивностью L = 0,5 Гн накапливается энергия WL = 6,25 Дж. Найти силу тока в котутци.

Решение. Из формулы WL = LI2 / 2 найдем величину тока:

Взаимоиндукция

Пусть имеем две катушки, расположенные рядом. Через первую катушку проходит ток, созданный источником ЭДС. Часть магнитных силовых линий этой катушки перетинае витки обеих катушек. Обозначим эту часть магнитного потока через Ф12. Если с помощью реостата начать изменять величину тока в цепи первой катушки, то такое изменение магнитного потока обусловливает индуцированной ЭДС, как в первой, так и во второй катушках.

ЭДС, возникающая в витках второй катушки от изменения магнитного потока первой катушки, называется ЭДС взаимоиндукции.

Явление возникновения индуцированной ЭДС в проводниках, расположенных вблизи других проводников, по которым проходит переменный во времени электрический ток, называется взаимоиндукции.

ЭДС взаимоиндукции образовываться также при замыкании и размыкании цепи первой катушки том, что при этом тоже происходит изменение тока и его магнитного потока от нуля до постоянной величины и от постоянной величины до нуля.

 
 

ЭДС самоиндукции во второй катушке определяется формулой
 
 

Изменение потокосцепления dΨ2 происходит за счет изменения тока di1, следовательно,
 
 

Тогда

Согласно этой формуле, величина ЭДС самоиндукции зависит от постоянных величин катушек и сердечника

N1N2 μa S/l

 
 

Обозначим это выражение буквой М:
 
 

Тогда

Коэффициент пропорциональности М называют взаимной индуктивности.

Итак, взаимной индуктивности и зиваеться величина, характеризующая степень передачи энергии от одной электрической цепи в другого.0диницею взаемоиндуктивности является генри.

 
 

Если круг второй обмотки включено на потребителя енергии, то ЭДС взаимной индукции eM2 создает в нем ток i2 магнитный поток Ф2, которые при

Таким образом, напряжение источника U1 частично падает на сопротивлении катушки R1, а частично идет на преодоление противодействия ЭДС самоиндукции eL1 и ЭДС взаемоиндукции EM1.

Связь между взаимной индуктивности M и индуктивностями катушек:

M2 = L1L2.

 
 

Отсюда
 
 

Это равенство выполняется при отсутствии магнитного рассеяния, то есть когда все магнитный поток, образованный током первой обмотки, проходит внутри второй обмотки, например, весь магнитный поток замыкается по ферромагнитного сердечника, вследствие его малого сопротивления магнитному потоку. При наличии магнитного рассеяния часть магнитного потока не проходит внутри второй катушки и ЭДС взаимоиндукции меньше, тогда это равенство надо брать с коэффициентом K

Коэффициент связи характеризует степень индуктивной связи двух контуров. Рассматривают три степени связи: очень слабый связь, K = 0,001 … 0,01; слабый связь, К = 0,01 … 0,1; сильная связь, К = 0,1 … 0,9.

При отсутствии индуктивной связи между катушками общая индуктивность цепи

L = L1+L2,

 
 

Для согласованного соединения катушек можно доказать, что при отсутствии индуктивной связи общая индуктивность

При наличии индуктивной связи при согласованном соединении катушек а при встречном соединении

 

Как видим, индуктивность взаимосвязанных катушек зависит от

коэффициента связи. На этом принципе построены приборы, предназначенные для постепенного изменения индуктивности, которые называются вариометрами. Вариометр имеет две последовательно соединенные катушки, одна из которых неподвижна, а другая – подвижная, малого размера и может вращаться внутри первой. При вращении подвижной катушки меняется взаимное положение катушек, поэтому меняется и коэффициент связи.

Когда магнитные потоки катушек имеют одинаковое направление, то индуктивность вариометра максимальная

Lmax = L1+L2+2M,

когда магнитные потоки имеют противоположные направления, то индуктивнисть вариометра наименьшая

Lmin = L1+L2-2M

Явление взаимоиндукции широко применяется в технике, на нем основана работа трансформаторов; индукционных катушек в системе зажигания автомашин и передачи энергии от одного индуктивно связанного контура ко второму, и тому подобное. Иногда взаимоиндукция бывает вредной. Например, образуются препятствия в линиях связи от внешних магнитных полей, мощных радиостанций, линий электропередач и т. Д. Магнитные поля индуцируют токи в линиях связи, которые мешают передачи сигналов связи.

Вихревые токи


Рекомендуемые страницы:


Воспользуйтесь поиском по сайту:

Электромагнитная индукция. – Закон электромагнитной индукции

Электромагнитная индукция — явление возникновения электрического тока в замкнутом контуре при изменении магнитного потока, проходящего через него.

Электромагнитная индукция была открыта Майклом Фарадеем 29 августа 1831 года. Он обнаружил, что электродвижущая сила, возникающая в замкнутом проводящем контуре, пропорциональна скорости изменения магнитного поток через поверхность, ограниченную этим контуром. Величина электродвижущей силы (ЭДС) не зависит от того, что является причиной изменения потока — изменение самого магнитного поля или движение контура (или его части) в магнитном поле. Электрический ток, вызванный этой ЭДС, называется индукционным током.

Зако́н электромагни́тной инду́кции Фараде́я является основным законом электродинамики, касающимся принципов работы трансформаторов,дросселей, многих видов электродвигателей и генераторов. Закон гласит:

  • Для любого замкнутого контура индуцированная электродвижущая сила (ЭДС) равна скорости изменения магнитного потока, проходящего через этот контур.

или другими словами:

  • Генерируемая ЭДС пропорциональна скорости изменения магнитного потока.

Согласно закону электромагнитной индукции Фарадея: 

где

 — электродвижущая сила, действующая вдоль произвольно выбранного контура,
  — магнитный поток через поверхность, натянутую на этот контур.

Знак «минус» в формуле отражает правило Ленца, названное так по имени русского физика Э. Х. Ленца:

Индукционный ток, возникающий в замкнутом проводящем контуре, имеет такое направление, что создаваемое им магнитное поле противодействует тому изменению магнитного потока, которым был вызван данный ток.

Для катушки, находящейся в переменном магнитном поле, закон Фарадея можно записать следующим образом:

где

 — электродвижущая сила,
 — число витков,
 — магнитный поток через один виток,
 — потокосцепление катушки.

Информация взята с http://ru.wikipedia.org/wiki/Закон_электромагнитной_индукции_

Электромагнитная индукция – white-santa.ru

 Электромагнитная индукция – это словосочетание  наводит на мысль о чем-то космическом и невесомом.  Конечно же электромагнитная индукция никак не связана с космосом или невесомостью, но все же какое-то подобие магии в этой области есть.

 

   Работа всех электрических машин основывается на явлении электромагнитной индукции. Иногда трудно себе представить, как такие машины способны работать.. Например, трансформатор – преобразует электрическую энергию одной величины в другую, при этом его обмотки не связаны друг с другом, фактически по воздуху.

   Асинхронные двигателя, их принцип действия также объясняет явление электромагнитной индукции. Какой силой обладает вращающийся ротор, какие механизмы он способен приводить в действие. Напомню, ротор так же ни с чем не связан, он свободно вращается на подшипниках вокруг своей оси.
Но от куда берется эта сила? Давайте копнем глубже и рассмотрим детально явление электромагнитной индукции.

Опыт Фарадея и электромагнитная индукция

   Для более глубокого понимания явления электромагнитной индукции давайте рассмотрим следующий опыт:

Между двух полюсов постоянного магнита расположим некий проводник. К концам этого проводника будет подключен гальванометр (чувствительный измерительный прибор).

   Обратим внимание. Когда проводник между полюсов магнита находится в состоянии покоя, стрелка прибора находится в среднем положении.

Стоит только переместить проводник, как стрелка тут же отклонится, при прекращении движения проводника стрелка проводника возвратится в среднее положение. Если проводник перемещать в обратном направлении, то стрелка прибора так же отклонится на время движения проводника, при этом направление отклонение стрелки гальванометра будит противоположным.

   Изменение положения стрелки гальванометра, в момент движения проводника в магнитном поле указывает на то, что в этом проводнике наводится некая электродвижущая сила (сокращенно э.д.с).
Появление этой силы, можно объяснить тем, что под действием магнитного поля, свободные электроны, находящиеся в проводнике, начинают упорядоченно двигаться по проводнику.

   Так как к нашему проводнику подключен измерительный прибор, то эта система из перемещаемого проводника и гальванометра с соединительными проводами представляет собой замкнутую цепь, а в этом случаи по цепи протекает электрический ток. На это и указывает стрелка гальванометра.

   Обратим внимание, что электрический ток, а ему предшествует наведение электродвижущей силы возникает лишь в момент движения проводника в магнитном поле постоянного магнита. А величина наведенной электродвижущей силы зависит от скорости перемещения проводника.

Закон электромагнитной индукции Фарадея

   Мы знаем, что величина наведенной электродвижущей силы в проводнике, движущемся в некотором магнитном поле, с определенной скоростью,  зависит от скорости передвижения проводника. Но это еще не все. Электродвижущая сила так же зависит от длины проводника, важна именно длина, которая находится под действием магнитного поля магнита. Еще зависит от индукции магнитного поля и от направления передвижения самого проводника.

М. Фарадей сформулировал закон электромагнитной индукции следующим образом:

«Индуцируемая электродвижущая сила прямо пропорциональна индукции магнитного поля B, длине проводника l и скорости его перемещения v в направлении, перпендикулярном силовым линиям поля.»

   Этот закон можно выразить формулой:

где электродвижущая сила обозначается буквой e:

Когда проводник движется не под прямым углом по отношению к магнитному полю  формула имеет следующий вид:

Где:
e – электродвижущая сила;

B – индукция магнитного поля;

l – длина проводника;

v – скорость перемещения проводника в магнитном поле;
Sin ϕ – синус угла под которым производится перемещение относительно магнитного поля.

   Индуцирование электродвижущей силы в проводнике происходит, когда он перемещается в магнитном поле. То есть пересечение магнитными силовыми линиями не должно быть постоянным, а всегда изменятся.
Электродвижущая сила в этом проводнике будит индуцироваться не зависимо от того, замкнута цепь проводника или нет.

   Для протекания электрического тока, основное условие — наличие замкнутой цепи, а для электродвижущей силы, главное условие ее наведения – это изменение силовых магнитных линий, пересекающих проводник.
   Заметьте, что движение проводника в магнитном поле не является основополагающим фактором индуцирования электродвижущей силы. Допускается и то, что проводник неподвижен, а перемещается лишь магнитное поле, в котором находится этот проводник.

Правило правой руки

Вы, наверное, обратили внимание, что при изменении направления перемещения проводника в магнитном поле изменяется и направление отклонения стрелочки гальванометра. Следовательно, и индуцируемая электродвижущая сила изменила свое направление.
Существует правило, благодаря которому можно определить направление индуцируемой электродвижущей силы. Это правило называется «Правило правой руки».

«Если ладонь правой руки держать так, чтобы в нее входили магнитные силовые линии поля, а отогнутый большой палец совместить с направлением движения проводника, то вытянутые четыре пальца укажут направление индуцированной электродвижущей силы»

Применение электромагнитной индукции

   Работа всех электрических машин переменного тока основывается на явлении электромагнитной индукции.

   Электромагнитная индукция  — серьёзное основание (база), понимание и овладение которым, открывает большинство дверей в мире электрических машин.
К таким машинам относят всем давно известные трансформаторы, электрические двигатели и  генераторы.
   Многие, наверное, слышали о индукционных печах, индукционный способ плавки, а индукционные счетчики электрической энергии уже устаревшие.
Принцип работы многих электрических аппаратов основывается на явлении магнитной индукции, это такие как магнитные пускатели, контакторы, различные типы реле и современные датчики положения.
   В современной технике данное явление применяется в беспроводных зарядках для телефонов, в микроволновых печах и так далее.

   Но существует и обратная сторона медали. Из-за явления электромагнитной индукции в электроэнергетике существуют колоссальные потери. На всем известные вихревые токи, которые наводятся практически везде. Хотя с этим видом потерь активно борются и находят те или иные способы уменьшения таких потерь,  все же они вещественны и ощутимы.

Закон электромагнитной индукции., калькулятор онлайн, конвертер

Законы электромагнитной индукции

Сущность электромагнитной индукции определяется замкнутым контуром с электропроводностью, площадь которого пропускает через себя изменяющийся магнитный поток. В этот момент под влиянием магнитного потока появляется электродвижущая сила Еi и в контуре начинает течь электрический ток.

Закон Фарадея для электромагнитной индукции заключается в прямой зависимости ЭДС и скорости, составляющих пропорцию. Данная скорость представляет собой время, в течение которого магнитный поток подвергается изменениям.

Данный закон выражается формулой Еi = – ∆Ф/∆t, в которой Еi – значение электродвижущей силы, возникающей в контуре, а ∆Ф/∆t является скоростью изменения магнитного потока. В этой формуле не совсем понятным остается знак «минус», но ему тоже имеется свое объяснение. В соответствии с правилом русского ученого Ленца, изучавшего открытия Фарадея, этот знак отображает направление ЭДС, возникающей в контуре. То есть, направление индукционного тока происходит таким образом, что создаваемый им магнитный поток на площади, ограниченной контуром, препятствует изменениям, вызванным этим током.

Открытия Фарадея были доработаны Максвеллом, у которого теория электромагнитного поля получила новые направления. В результате, появился закон Фарадея и Максвелла, выраженный в следующих формулах:

  • Edl = -∆Ф/∆t – отображает электродвижущую силу.
  • Hdl = -∆N/∆t – отображает магнитодвижущую силу.

В этих формулах Е соответствует напряженности электрического поля на определенном участке dl, Н является напряженностью магнитного поля на этом же участке, N – поток электрической индукции, t – период времени.

Оба уравнения отличаются симметричностью, позволяющей сделать вывод, что магнитные и электрические явления связаны между собой. С физической точки зрения эти формулы определяют следующее:

  • Изменениям в электрическом поле всегда сопутствует образование магнитного поля.
  • Изменения в магнитном поле всегда происходят одновременно с образованием электрического поля.

Изменяющийся магнитный поток, проходящий сквозь замкнутую конфигурацию проводящего контура, приводит к возникновению в этом контуре электрического тока. Это основная формулировка закона Фарадея. Если изготовить проволочную рамку и поместить ее внутри вращающегося магнита, то в самой рамке появится электричество.

Это и будет индукционный ток, в полном соответствии с теорией и законом Майкла Фарадея. Изменения магнитного потока, проходящего через контур, могут быть произвольными. Следовательно, формула ∆Ф/∆t бывает не только линейной, а в определенных условиях принимает любую конфигурацию. Если изменения происходят линейно, то ЭДС электромагнитной индукции, возникающей в контуре, будет постоянной. Временной интервал t становится каким угодно, а отношение ∆Ф/∆t не будет зависеть от его продолжительности.

Если же изменения магнитного потока принимают более сложную форму, то ЭДС индукции уже не будет постоянной, а будет зависеть от данного промежутка времени. В этом случае временной интервал рассматривается в качестве бесконечно малой величины и тогда соотношение ∆Ф/∆t с точки зрения математики станет производной от изменяющегося магнитного потока.

Существует еще один вариант, трактующий закон электромагнитной индукции Фарадея. Его краткая формулировка объясняет, что действие переменного магнитного поля вызывает появление вихревого электрического поля. Этот же закон можно трактовать как одну из характеристик электромагнитного поля: вектор напряженности поля может циркулировать по любому из контуров со скоростью, равной скорости изменения магнитного потока, проходящего через тот или иной контур.

Закон электромагнитной индукции формула

Закон Фарадея для электролиза

Индукция магнитного поля

Закон полного тока

Клетка Фарадея

Закон Ома для полной цепи

История

Электромагнитная индукция была обнаружена независимо друг от друга Майклом Фарадеем и Джозефом Генри в 1831 году, однако Фарадей первым опубликовал результаты своих экспериментов.

В первой экспериментальной демонстрации электромагнитной индукции (август 1831) Фарадей обмотал двумя проводами противоположные стороны железного тора (конструкция похожа на современный трансформатор). Основываясь на своей оценке недавно обнаруженного свойства электромагнита, он ожидал, что при включении тока в одном проводе особого рода волна пройдёт сквозь тор и вызовет некоторое электрическое влияние на его противоположной стороне. Он подключил один провод к гальванометру и смотрел на него, когда другой провод подключал к батарее. В самом деле, он увидел кратковременный всплеск тока (который он назвал «волной электричества»), когда подключал провод к батарее, и другой такой же всплеск, когда отключал его. В течение двух месяцев Фарадей нашёл несколько других проявлений электромагнитной индукции. Например, он увидел всплески тока, когда быстро вставлял магнит в катушку и вытаскивал его обратно, он генерировал постоянный ток во вращающемся вблизи магнита медном диске со скользящим электрическим проводом («диск Фарадея»).

Диск Фарадея

Фарадей объяснил электромагнитную индукцию с использованием концепции так называемых силовых линий. Однако, большинство учёных того времени отклонили его теоретические идеи, в основном потому, что они не были сформулированы математически. Исключение составил Максвелл, который использовал идеи Фарадея в качестве основы для своей количественной электромагнитной теории. В работах Максвелла аспект изменения во времени электромагнитной индукции выражен в виде дифференциальных уравнений. Оливер Хевисайд назвал это законом Фарадея, хотя он несколько отличается по форме от первоначального варианта закона Фарадея и не учитывает индуцирование ЭДС при движении. Версия Хевисайда является формой признанной сегодня группы уравнений, известных как уравнения Максвелла.

Эмилий Христианович Ленц сформулировал в 1834 году закон (правило Ленца), который описывает «поток через цепь» и даёт направление индуцированной ЭДС и тока в результате электромагнитной индукции.

Эксперимент Фарадея, показывающий индукцию между витками провода: жидкостная батарея (справа) даёт ток, который протекает через небольшую катушку (A), создавая магнитное поле. Когда катушки неподвижны, ток не индуцируется. Но когда маленькая катушка вставляется или извлекается из большой катушки (B), магнитный поток через катушку изменяется, вызывая ток, который регистрируется гальванометром (G).

История развития

После доказательства закона электромагнитной индукции английским ученым М. Фарадеем над открытием работали российские ученые Э. Ленц и Б. Якоби. Благодаря их трудам, сегодня разработанный принцип положен в основу функционирования многих приборов и механизмов.

Основными агрегатами, в которых применяется закон электромагнитной индукции Фарадея, являются двигатель, трансформатор и множество иных приборов.

Индукцией электромагнитно именуется индуцирование в замкнутой проводящей системе электрического тока. Такое явление становится возможным при физическом передвижении через проводниковую систему магнитного поля. Механическое действие влечет за собой появление электричества. Его принято называть индукционным. До открытия закона Фарадея человечество не знало об иных способах создания электричества, кроме гальваники.

Если сквозь проводник пропустить магнитное поле, в нем будет возникать ЭДС индукции. Ее еще именуют электродвижущей силой. При помощи этого открытия удается представить в количественном выражении показатель.

Электрический генератор

Рис. 8. Электрический генератор на основе диска Фарадея. Диск вращается с угловой скоростью ω, при этом проводник, расположенный вдоль радиуса, движется в статическом магнитном поле B. Магнитная сила Лоренца v × B создаёт ток вдоль проводника по направлению к ободу, затем цепь замыкается через нижнюю щётку и ось поддержки диска. Таким образом, вследствие механического движения генерируется ток.

Явление возникновения ЭДС, порождённой по закону индукции Фарадея из-за относительного движения контура и магнитного поля, лежит в основе работы электрических генераторов. Если постоянный магнит перемещается относительно проводника или наоборот, проводник перемещается относительно магнита, то возникает электродвижущая сила. Если проводник подключён к электрической нагрузке, то через неё будет течь ток, и следовательно, механическая энергия движения будет превращаться в электрическую энергию. Например, дисковый генератор построен по тому же принципу, как изображено на рис. 4. Другой реализацией этой идеи является диск Фарадея, показанный в упрощённом виде на рис. 8

Обратите внимание, что и анализ рис. 5, и прямое применение закона силы Лоренца показывают, что твёрдый проводящий диск работает одинаковым образом.

В примере диска Фарадея диск вращается в однородном магнитном поле, перпендикулярном диску, в результате чего возникает ток в радиальном плече благодаря силе Лоренца. Интересно понять, как получается, что чтобы управлять этим током, необходима механическая работа. Когда генерируемый ток течёт через проводящий обод, по закону Ампера этот ток создаёт магнитное поле (на рис. 8 оно подписано «индуцированное B» — Induced B). Обод, таким образом, становится электромагнитом, который сопротивляется вращению диска (пример правила Ленца). В дальней части рисунка обратный ток течёт от вращающегося плеча через дальнюю сторону обода к нижней щётке. Поле В, создаваемое этим обратным током, противоположно приложенному полю, вызывая сокращение потока через дальнюю сторону цепи, в противовес увеличению потока, вызванного вращением. На ближней стороне рисунка обратный ток течёт от вращающегося плеча через ближнюю сторону обода к нижней щётке. Индуцированное поле B увеличивает поток по эту сторону цепи, в противовес снижению потока, вызванного вращением. Таким образом, обе стороны цепи генерируют ЭДС, препятствующую вращению. Энергия, необходимая для поддержания движения диска в противовес этой реактивной силе, в точности равна вырабатываемой электрической энергии (плюс энергия на компенсацию потерь из-за трения, из-за выделения тепла Джоуля и прочее). Такое поведение является общим для всех генераторов преобразования механической энергии в электрическую.

Хотя закон Фарадея описывает работу любых электрических генераторов, детальный механизм в разных случаях может отличаться. Когда магнит вращается вокруг неподвижного проводника, меняющееся магнитное поле создаёт электрическое поле, как описано в уравнении Максвелла-Фарадея, и это электрическое поле толкает заряды через проводник. Этот случай называется индуцированной ЭДС. С другой стороны, когда магнит неподвижен, а проводник вращается, на движущиеся заряды воздействует магнитная сила (как описывается законом Лоренца), и эта магнитная сила толкает заряды через проводник. Этот случай называется двигательной ЭДС.

Математический вид

Законы Фарадея можно записать в виде следующей формулы:

m = (QF)(Mz),{\displaystyle m\ =\ \left({Q \over F}\right)\left({M \over z}\right),}

где:

  • m{\displaystyle m} — масса осаждённого на электроде вещества,
  • Q{\displaystyle Q} — полный электрический заряд, прошедший через вещество
  • F=96485,33(83){\displaystyle F=96\,485,33(83)} Кл·моль−1 — постоянная Фарадея,
  • M{\displaystyle M}— молярная масса вещества (Например, молярная масса воды h3O{\displaystyle {\ce {h3O}}} = 18 г/моль),
  • z{\displaystyle z} — валентное число ионов вещества (число электронов на один ион).

Заметим, что Mz{\displaystyle M/z} — это эквивалентная масса осаждённого вещества.

Для первого закона Фарадея M,F{\displaystyle M,\,F} и z{\displaystyle z} являются константами, так что, чем больше величина Q{\displaystyle Q}, тем больше будет величина m{\displaystyle m}.

Для второго закона Фарадея Q,F{\displaystyle Q,\,F} и z{\displaystyle z} являются константами, так что чем больше величина Mz{\displaystyle M/z} (эквивалентная масса), тем больше будет величина m{\displaystyle m}.

В простейшем случае используется постоянный ток и полный электрический заряд (прошедший через систему) за время электролиза равен: Q=It{\displaystyle Q=It} , что приводит к выражению:

m = (ItF)(Mz),{\displaystyle m\ =\ \left({It \over F}\right)\left({M \over z}\right),} где размерность тока I{\displaystyle I} ампер-час (ампер-секунда и др.) определяет размерность времени электролиза t{\displaystyle t}.

и тогда

n = (ItF)(1z),{\displaystyle n\ =\ \left({It \over F}\right)\left({1 \over z}\right),}

где:

  • n{\displaystyle n} — выделенное количество вещества («количество молей»): n=mM{\displaystyle n=m/M},
  • t{\displaystyle t} — время действия постоянного тока.

В более сложном случае переменного электрического тока полный заряд Q{\displaystyle Q} тока I(τ){\displaystyle I(\tau )} суммируется за время τ{\displaystyle \tau }:

Q=∫tI(τ) dτ.{t}I(\tau )\ d\tau .}

Здесь t{\displaystyle t} — полное время электролиза, τ{\displaystyle \tau } переменная времени, ток I{\displaystyle I} является функцией от времени τ{\displaystyle \tau }.

История развития

После доказательства закона электромагнитной индукции английским ученым М. Фарадеем над открытием работали российские ученые Э. Ленц и Б. Якоби. Благодаря их трудам, сегодня разработанный принцип положен в основу функционирования многих приборов и механизмов.

Индукцией электромагнитно именуется индуцирование в замкнутой проводящей системе электрического тока. Такое явление становится возможным при физическом передвижении через проводниковую систему магнитного поля. Механическое действие влечет за собой появление электричества. Его принято называть индукционным. До открытия закона Фарадея человечество не знало об иных способах создания электричества, кроме гальваники.

Если сквозь проводник пропустить магнитное поле, в нем будет возникать ЭДС индукции. Ее еще именуют электродвижущей силой. При помощи этого открытия удается представить в количественном выражении показатель.

При изменении магнитного потока через замкнутый проводящий контур, в контуре возникает электрический ток. То есть, если мы скрутим из проволоки рамку и поместим ее в изменяющееся магнитное поле (возьмем магнит, и будем крутить его вокруг рамки), по рамке потечет ток!

Рамка в поле

Этот ток Фарадей назвал индукционным, а само явление окрестил электромагнитной индукцией.

А откуда в формуле минус, спросите Вы. Для объяснения знака минус в этой формуле есть специальное правило Ленца. Оно гласит, что знак минус, в данном случае, указывает на то, как направлена возникающая ЭДС. Дело в том, что создаваемое индукционным током магнитное поле направлено так, что препятствует изменению магнитного потока, который вызвал индукционный ток.

Правило правой руки

Опытное доказательство

Проводя свои исследования, английский ученый установил, что индукционный ток получается одним из двух способов. В первом опыте он появляется при движении рамки в магнитном поле, создаваемом неподвижной катушкой. Второй способ предполагает неподвижное положение рамки. В этом эксперименте изменяется только поле катушки при ее движении или изменении силы тока в ней.

Опыты Фарадея привели исследователя к выводу, что при генерировании индукционного тока провоцируется увеличением или уменьшением магнитного потока в системе. Также опыты Фарадея позволили утверждать, что значение электричества, полученного опытным путем, не зависит от методологии, которой был изменен поток магнитной индукции. На показатель влияет только скорость такого изменения.

Закон Фарадея как два различных явления

Некоторые физики отмечают, что закон Фарадея в одном уравнении описывает два разных явления: двигательную ЭДС, генерируемую действием магнитной силы на движущийся провод, и трансформаторную ЭДС, генерируемую действием электрической силы вследствие изменения магнитного поля

Джеймс Клерк Максвелл обратил внимание на этот факт в своей работе О физических силовых линиях в 1861 году. Во второй половине части II этого труда Максвелл даёт отдельное физическое объяснение для каждого из этих двух явлений

Ссылка на эти два аспекта электромагнитной индукции имеется в некоторых современных учебниках. Как пишет Ричард Фейнман:

Отражение этой очевидной дихотомии было одним из основных путей, которые привели Эйнштейна к разработке специальной теории относительности:

Паразитная индукция и тепловые потери

В любом металлическом объекте, движущемся по отношению к статическому магнитному полю, будут возникать индукционные токи, как и в любом неподвижном металлическом предмете по отношению к движущемуся магнитному полю. Эти энергетические потоки в сердечниках трансформаторов нежелательны, из-за них в слое металла течёт электрический ток, который нагревает металл.

В соответствии с правилом Ленца вихревые токи протекают внутри проводника по таким путям и направлениям, чтобы своим действием возможно сильнее противится причине, которая их вызывает. Вследствие этого при движении в магнитном поле на хорошие проводники действует тормозящая сила, вызываемая взаимодействием вихревых токов с магнитным полем. Этот эффект используется в ряде приборов для демпфирования колебаний их подвижных частей.

Есть ряд методов, используемых для борьбы с этими нежелательными индуктивными эффектами.

  • Электромагниты в электрических двигателях, генераторах и трансформаторах не делают из сплошного металла, а используют тонкие листы жести, называемые «ламинатами». Эти тонкие пластины уменьшают паразитные вихревые токи, как будет описано ниже.
  • Катушки индуктивности в электронике обычно используют магнитные сердечники, чтобы минимизировать паразитный ток. Их делают из смеси металлического порошка со связующим наполнителем, и они имеют различную форму. Связующий материал предотвращает прохождение паразитных токов через порошковый металл.

Расслоение электромагнита


Вихревые токи возникают, когда сплошная масса металла вращается в магнитном поле, так как внешняя часть металла пересекает больше силовых линий, чем внутренняя, следовательно, индуцированная электродвижущая сила неравномерна и стремится создать токи между точками с наибольшим и наименьшим потенциалами. Вихревые токи потребляют значительное количество энергии, и часто приводят к вредному повышению температуры.

На этом примере показаны всего пять ламинатов или пластин для демонстрации расщепление вихревых токов. На практике число пластин или перфорация составляет от 40 до 66 на дюйм, что приводит к снижению потерь на вихревых токах примерно до одного процента. Хотя пластины могут быть отделены друг от друга изоляцией, но поскольку возникающие напряжения чрезвычайно низки, то естественной ржавчины или оксидного покрытия пластин достаточно, чтобы предотвратить ток через пластины.

Это ротор от двигателя постоянного тока диаметром примерно 20 мм, используемого в проигрывателях компакт-дисков

Обратите внимание, для снижения паразитных индуктивных потерь сделано расслоение полюса электромагнита на части.

Паразитные потери в катушках индуктивности


На этой иллюстрации сплошной медный стержень катушки индуктивности во вращающемся якоре просто проходит под кончиком полюса N магнита

Обратите внимание на неравномерное распределение силовых линий через стержень. Магнитное поле имеет большую концентрацию и, следовательно, сильнее на левом краю медного стержня (a, b), тогда как слабее по правому краю (c, d)

Поскольку два края стержня будут двигаться с одинаковой скоростью, это различие в напряженности поля через стержень создаст вихри тока внутри медного стержня.

Это одна из причин, по которой устройства с высоким напряжением, как правило, более эффективны, чем низковольтные устройства. Высоковольтные устройства имеют множество небольших витков провода в двигателях, генераторах и трансформаторах. Эти многочисленные небольшие витки провода в электромагните разбивают вихревые потоки, а в пределах больших, толстых катушек индуктивности низкого напряжения образуется вихревые токи большей величины.

Примечания

  1. , с. 208.
  2. Michael Faraday, by L. Pearce Williams, p. 182-3
  3. Michael Faraday, by L. Pearce Williams, p. 191-5
  4. Michael Faraday, by L. Pearce Williams, p. 510
  5. Maxwell, James Clerk (1904), A Treatise on Electricity and Magnetism, Vol. II, Third Edition. Oxford University Press, pp. 178-9 and 189.
  6. В-поле наведенного тока ведет к снижению магнитного потока, в то время как движение цикла имеет тенденцию к увеличению (так как В (х) возрастает по мере цикла движений). Эти противоположные действия — пример принципа Ле Шателье в форме закона Ленца.
  7. K. Simonyi, Theoretische Elektrotechnik, 5th edition, VEB Deutscher Verlag der Wissenschaften, Berlin 1973, equation 20, page 47
  8. В этом примере предполагается, что скорости движения намного меньше скорости света, поэтому корректировкой поля, связанной с преобразованиями Лоренца, можно пренебречь.
  9. Единственным способом определения этого является измерение x от xC в движущемся контуре, скажем ξ = x — xC (t). Тогда за время t движущийся наблюдатель увидит поле B (ξ, t), тогда как неподвижный наблюдатель увидит в той же точке поле B [ ξ + xC (t) ] = B (ξ + xC0 + v t) при xC0 = xC (t = 0).
  10. Images and reference text are from the public domain book: Hawkins Electrical Guide, Volume 1, Chapter 19: Theory of the Armature, pp. 272—273, Copyright 1917 by Theo. Audel & Co., Printed in the United States
  11. Images and reference text are from the public domain book: Hawkins Electrical Guide, Volume 1, Chapter 19: Theory of the Armature, pp. 270—271, Copyright 1917 by Theo. Audel & Co., Printed in the United States

Электродинамика

Первые работы применяются в физике, конкретно в описании работы электрических машин и аппаратов (трансформаторов, двигателей и пр.). Закон Фарадея гласит:

Для контура индуцированная ЭДС прямо пропорциональна величине скорости магнитного потока, который перемещается через этот контур со знаком минус.

Это можно сказать простыми словами: чем быстрее магнитный поток движется через контур, тем больше на его выводах генерируется ЭДС.

Формула выглядит следующим образом:

Здесь dФ – магнитный поток, а dt – единица времени. Известно, что первая производная по времени – это скорость. Т.е скорость перемещения магнитного потока в данном конкретном случае. Кстати перемещаться может, как и источник магнитного поля (катушка с током – электромагнит, или постоянный магнит), так и контур.

Здесь же поток можно выразить по такой формуле:

B – магнитное поле, а dS – площадь поверхности.

Если рассматривать катушку с плотнонамотанными витками, при этом в количестве витков N, то закон Фарадея выглядит следующим образом:

Магнитный поток в формуле на один виток, измеряется в Веберах. Ток, протекающий в контуре, называется индукционным.

Электромагнитная индукция – явление протекания тока в замкнутом контуре под воздействием внешнего магнитного поля.

В формулах выше вы могли заметить знаки модуля, без них она имеет слегка иной вид, такой как было сказано в первой формулировке, со знаком минус.

Знак минус объясняет правило Ленца. Ток, возникающий в контуре, создает магнитное поле, оно направлено противоположно. Это является следствием закона сохранения энергии.

Направление индукционного тока можно определить по правилу правой руки или буравчика, мы его рассматривали на нашем сайте подробно.

Как уже было сказано, благодаря явлению электромагнитной индукции работают электрические машины трансформаторы, генераторы и двигатели. На иллюстрации показано протекание тока в обмотке якоря под воздействием магнитного поля статора. В случае с генератором, при вращении его ротора внешними силами в обмотках ротора возникает ЭДС, ток порождает магнитное поле направленное противоположно (тот самый знак минус в формуле). Чем больше ток, потребляемый нагрузкой генератора, тем больше это магнитное поле, и тем больше затрудняется его вращение.

И наоборот — при протекании тока в роторе возникает поле, которое взаимодействует с полем статора и ротор начинает вращаться. При нагрузке на вал ток в статоре и в роторе повышается, при этом нужно обеспечить переключение обмоток, но это уже другая тема, связанная с устройством электрических машин.

В основе работы трансформатора источником движущегося магнитного потока является переменное магнитное поле, возникающее в следствие протекания в первичной обмотке переменного тока.

Если вы желаете более подробно изучить вопрос, рекомендуем просмотреть видео, на котором легко и доступно рассказывается Закон Фарадея для электромагнитной индукции:

Работа Э. Ленца

Направленность индукционного тока предоставляет возможность определить правило Ленца. Краткая формулировка звучит достаточно просто. Появляющийся при изменении показателей поля проводникового контура ток, препятствует благодаря своему магнитному полю такому изменению.

Если магнит выводится из системы, магнитный поток в ней уменьшится. Чтобы установить направление тока, выкручивается буравчик. Вращения будет направлено в обратную сторону перемещения по циферблату часовой стрелки.

Формулировки Ленца приобретают большое значение для системы с контуром замкнутого типа и отсутствующим сопротивлением. Его принято именовать идеальным контуром. По правилу Ленца, в нем невозможно увеличить или уменьшить магнитный поток.

Индуцированная проблема ЭДС

Возьмем простейший случай однородного магнитного поля и круглого провода, перпендикулярного магнитному полю. Вы знаете, что изменяющееся магнитное поле вызывает электрическое поле. Поскольку магнитное поле однородно, а провод перпендикулярен, электрическое поле имеет одинаковую величину во всех точках провода. Вы можете найти общий ЭДС от изменения потока. Но помните определение ЭДС для замкнутого цикла.

е м ф= ∮СЕ⃗ ⋅ дLемезнак равно∮СЕ→⋅dL

Это также равно – dΦВdT-dΦВdTдля того же замкнутого цикла. Теперь предположим, что вы рассчитываете по магнитному потоку, что индуцированная ЭДС составляет 6 В. Это также означает взятие линейного интеграла от E⃗ Е→поле вокруг всей замкнутой петли дает вам 6V. Теперь вместо того, чтобы обходить весь цикл, интегрируйте около половины цикла. Наш E⃗ Е→Поле имеет одинаковую величину в каждой точке, поэтому мы получаем половину ЭДС, 3В. Если мы возьмем две точки, которые составляют всего 10% от общей окружности петли, то между этими двумя точками будет эдс в 6,6 В.

Другими словами, в отличие от батареи, в которой эдс ограничена одной точкой на схеме, в случае индуцированной эдс имеется усиление напряжения на любых двух точках контура, где ∫ВAЕ⃗ ⋅ дL∫AВЕ→⋅dL ненулевой

Чтобы затем найти разность напряжений между двумя точками, необходимо добавить вклады ∫ВAЕ⃗ ⋅ дL∫AВЕ→⋅dLи от резисторов. Вы знаете, что индуцированная ЭДС равна падению напряжения на резисторах. Таким образом, если эдс 6В, напряжение на обоих резисторах -6В. Давайте, скажем, снова используем простейший пример, где поле имеет одинаковую величину в каждой точке. Выберите две точки, чтобы у них было два резистора и 25% провода между ними. В части петли без резисторов и 75% проволочной петли вы увидите усиление 4,5 В. В секции с 25% провода и обоими резисторами вы увидите усиление 1,5 В – 6 В = -4,5 В.

Абсолютное напряжение в любой точке будет трудно рассчитать без контрольной точки. По крайней мере, вам нужно знать, был ли провод изначально на 0 В или на каком-либо другом В0В0и тогда я думаю, что вы сможете рассчитать это. Но обычно в таких случаях вы заземлите часть цепи.

Электромагнетизм – Индуцированная ЭДС в генераторе переменного тока

Генератор переменного тока: – Электрическое устройство, преобразующее механическую энергию в электрическую.

Принцип: Закон электромагнитной индукции Фарадея.

Строительство: Генератор переменного тока состоит из, Арматура. Катушка. Магнитные полюса. Контактные кольца. Угольные щетки.

Арматура: Ряд катушек намотан на цилиндрический сердечник, называемый якорем.

Катушка

: Он имеет прямоугольную форму и способен вращаться вокруг своей оси в магнитном поле.

Магнитные полюса: Есть два северных и южных полюса для создания однородного магнитного поля в области вращения катушки.

Контактные кольца: Есть два круглых кольца, каждое из которых связано с одним из выводов катушки. Они скользят по угольным щеткам.

Угольные щетки: Они используются для передачи тока от катушки к внешней цепи.

Рабочие: Если катушку вращать в магнитном поле, в катушке будет индуцироваться ток. Сила этого индуцированного тока зависит от магнитного потока.Магнитный поток максимален, когда плоскость катушки параллельна магнитному полю. Магнитный поток минимален, когда плоскость катушки перпендикулярна магнитному полю. Таким образом, когда катушка вращается в магнитном поле, индуцированный ток в ней непрерывно изменяется от максимального до минимального значения и от минимального до максимального значения и так далее. Якорь устроен так, что он может свободно вращаться в магнитном поле. Когда якорь поворачивается, то из-за изменения магнитного потока индуцированная эл.м.ф.

Факторы, от которых зависит ЭДС: Сила ЭДС может быть увеличена на: Увеличение количества витков катушки. Увеличение скорости катушки. Увеличение магнитного поля. Увеличение длины катушки. Увеличение угла между «V» и «B». ЭДС максимальна при угле 90 °.

Ток от генератора: Когда генератор включен по замкнутой цепи, наведенная ЭДС генерирует электрический ток. По мере вращения петель сила тока изменяется, как показано на графике.

  Ток минимален при θ = 0 °, плоскость контура перпендикулярна магнитному полю.
  

Ток максимален при θ = 90 °, плоскость контура параллельна магнитному полю.

Ток минимален при θ = 180 °, плоскость контура снова перпендикулярна магнитному полю.

Ток максимальный (отрицательный) при θ = 270 °, плоскость контура параллельна магнитному полю. то есть направление тока обратное.

Ток минимален при θ = 360 °, плоскость контура перпендикулярна магнитному полю.

Следует отметить, что ток увеличивается дважды за цикл, но в противоположном направлении. Ток и ЭДС плавно изменяются от нуля до максимального значения и обратно до нуля в течение каждой половины оборота контура.

Электромагнетизм – Проблема наведенной ЭДС

Возьмем простейший случай однородного магнитного поля и круглого провода, перпендикулярного магнитному полю. Вы знаете, что изменяющееся магнитное поле индуцирует электрическое поле. Поскольку магнитное поле однородно, а провод перпендикулярен, электрическое поле имеет одинаковую величину во всех точках провода.Вы можете найти полную ЭДС по изменению потока. Но помните определение ЭДС для замкнутого контура. $$ emf = \ oint_C \ vec {E} \ cdot dl $$ Это также равно $ – \ frac {d \ Phi_B} {dt} $ для того же замкнутого цикла. Теперь предположим, что вы рассчитали по магнитному потоку, что наведенная ЭДС составляет 6 В. Это также означает, что если взять линейный интеграл поля $ \ vec {E} $ вокруг всего замкнутого контура, получится 6V. Теперь вместо того, чтобы обходить весь цикл, объедините примерно половину цикла. Наше поле $ \ vec {E} $ имеет одинаковую величину во всех точках, поэтому мы получаем половину ЭДС, 3В.B \ vec {E} \ cdot dl $ и от резисторов. Вы знаете, что наведенная ЭДС равна падению напряжения на резисторах. Таким образом, если ЭДС составляет 6 В, напряжение на обоих резисторах составляет -6 В. Итак, давайте снова воспользуемся простейшим примером, где поле имеет одинаковую величину в каждой точке. Выберите две точки так, чтобы между ними было два резистора и 25% провода. В части контура без резисторов и на 75% проволочного контура вы увидите усиление 4,5 В. На участке с 25% провода и обоими резисторами вы увидите 1.5 В – 6 В = усиление -4,5 В.

Абсолютное напряжение в любой точке было бы трудно вычислить без контрольной точки. Вам, по крайней мере, нужно знать, был ли провод изначально на 0 В или какой-то другой $ V_0 $, и тогда, я думаю, вы сможете его вычислить. Но обычно в таких случаях вы заземляете часть цепи.

Motional Emf – Университетская физика, том 2

Цели обучения

К концу этого раздела вы сможете:

  • Определить величину наведенной ЭДС в проводе, движущемся с постоянной скоростью через магнитное поле.
  • Обсудите примеры, в которых используется ЭДС движения, например, рельсовая пушка и привязанный спутник.

Магнитный поток зависит от трех факторов: силы магнитного поля, площади, через которую проходят силовые линии, и ориентации поля с площадью поверхности.Если какая-либо из этих величин изменяется, происходит соответствующее изменение магнитного потока. До сих пор мы рассматривали только изменения потока из-за изменяющегося поля. Теперь мы рассмотрим другую возможность: изменение области, через которую проходят силовые линии, включая изменение ориентации области.

Два примера этого типа изменения потока представлены на (Рисунок). В части (а) поток через прямоугольную петлю увеличивается по мере того, как она движется в магнитное поле, а в части (b) поток через вращающуюся катушку изменяется в зависимости от угла.

(a) Магнитный поток изменяется, когда петля движется в магнитное поле; (б) магнитный поток изменяется при вращении петли в магнитном поле.

Интересно отметить, что то, что мы воспринимаем как причину определенного изменения потока, на самом деле зависит от выбранной нами системы отсчета. Например, если вы находитесь в состоянии покоя относительно движущихся катушек на (Рисунок), вы увидите, что поток изменяется из-за изменения магнитного поля – в части (а) поле перемещается слева направо в вашей системе отсчета, и в части (б) поле вращается.Часто можно описать изменение магнитного потока через катушку, которая движется в одной конкретной системе отсчета, в терминах изменяющегося магнитного поля во второй системе отсчета, где катушка неподвижна. Однако вопросы системы отсчета, связанные с магнитным потоком, выходят за рамки этого учебника. Мы избежим таких сложностей, всегда работая в кадре в состоянии покоя относительно лаборатории и объясняя вариации потока как следствие либо изменяющегося поля, либо изменяющейся области.

Теперь давайте посмотрим на проводящий стержень, включенный в цепь, изменяющую магнитный поток.Площадь, ограниченная схемой «MNOP» (рисунок), составляет лк и перпендикулярна магнитному полю, поэтому мы можем упростить интеграцию (рисунок) в умножение магнитного поля и площади. Следовательно, магнитный поток через открытую поверхность составляет

.

Так как B и l постоянны, а скорость стержня равна, теперь мы можем переформулировать закон Фарадея (рисунок) для величины ЭДС в единицах движущегося проводящего стержня как

Ток, наведенный в цепи, равен ЭДС, деленной на сопротивление, или

Кроме того, направление индуцированной ЭДС удовлетворяет закону Ленца, что вы можете проверить, посмотрев на рисунок.

Этот расчет ЭДС, вызванной движением, не ограничивается перемещением стержня по проводящим рельсам. В качестве отправной точки можно показать, что справедливо для любого изменения магнитного потока, вызванного движением проводника. Мы видели в законе Фарадея, что ЭДС, индуцированная изменяющимся во времени магнитным полем, подчиняется той же зависимости, которая является законом Фарадея. Таким образом, закон Фарадея выполняется для всех изменений магнитного потока , независимо от того, вызваны ли они изменяющимся магнитным полем, движением или их комбинацией.

Проводящий стержень перемещается вправо с постоянной скоростью. Результирующее изменение магнитного потока вызывает в цепи ток.

С точки зрения энергии производит мощность, а резистор ее рассеивает. Поскольку стержень движется с постоянной скоростью, приложенная сила должна уравновешивать магнитную силу на стержне, когда он пропускает индуцированный ток I . Таким образом, произведенная мощность составляет

ед.

Рассеиваемая мощность

В соответствии с принципом сохранения энергии производимая и рассеиваемая мощности равны.

Этот принцип можно увидеть в работе рельсового пистолета. Рельсовая пушка – это электромагнитная пусковая установка для снарядов, в которой используется устройство, подобное (Рисунок), и схематически оно показано на (Рисунок). Проводящий стержень заменяется выстрелом или оружием. До сих пор мы слышали только о том, как движение вызывает ЭДС. В рельсовой пушке оптимальное отключение / снижение магнитного поля уменьшает поток между рельсами, вызывая протекание тока в стержне (якорь), удерживающем снаряд.Этот ток через якорь испытывает магнитную силу и продвигается вперед. Однако рельсовые пушки не используются широко в вооруженных силах из-за высокой стоимости производства и больших токов: для выработки энергии, достаточной для того, чтобы рельсовая пушка была эффективным оружием, требуется около миллиона ампер.

Ток через две рельсы движет токопроводящий снаряд вперед за счет создаваемой магнитной силы.

Мы можем вычислить ЭДС, индуцированную движением, с помощью закона Фарадея , даже когда фактически замкнутый контур отсутствует .Мы просто представляем замкнутую область, граница которой включает движущийся проводник, вычисляем, а затем находим ЭДС по закону Фарадея. Например, мы можем позволить движущемуся стержню (Рисунок) быть одной стороной воображаемой прямоугольной области, представленной пунктирными линиями. Площадь прямоугольника составляет лк , поэтому магнитный поток через него равен. Дифференцируя это уравнение, получаем

, что соответствует разности потенциалов между концами стержня, которую мы определили ранее.

С показанным воображаемым прямоугольником мы можем использовать закон Фарадея для расчета наведенной ЭДС в движущемся стержне.

ЭДС движения в слабом магнитном поле Земли обычно не очень велики, иначе мы могли бы заметить напряжение на металлических стержнях, таких как отвертка, во время обычных движений. Например, простой расчет ЭДС движения стержня длиной 1,0 м, движущегося со скоростью 3,0 м / с перпендикулярно полю Земли, дает

Это небольшое значение согласуется с опытом.Однако есть впечатляющее исключение. В 1992 и 1996 годах с космическим шаттлом были предприняты попытки создать большие двигательные ЭДС. Привязанный спутник должен был быть выпущен на проводе длиной 20 км, как показано на (Рисунок), для создания ЭДС 5 кВ за счет движения с орбитальной скоростью через поле Земли. Эту ЭДС можно было бы использовать для преобразования некоторой кинетической и потенциальной энергии шаттла в электрическую, если бы можно было создать полную схему. Чтобы замкнуть цепь, неподвижная ионосфера должна была обеспечить обратный путь, по которому мог течь ток.(Ионосфера – это разреженная и частично ионизированная атмосфера на орбитальных высотах. Она является проводящей из-за ионизации. Ионосфера выполняет ту же функцию, что и неподвижные рельсы и соединительный резистор на (Рисунок), без которых не было бы полной цепи.) Затягивание тока в кабеле из-за магнитной силы выполняет работу, которая уменьшает кинетическую и потенциальную энергию шаттла и позволяет преобразовывать ее в электрическую энергию. Оба теста не увенчались успехом. В первом случае кабель завис, и его можно было протянуть только на пару сотен метров; во втором трос оборвался при почти полном растяжении.(Рисунок) указывает на выполнимость в принципе.

ЭДС движения как преобразование электроэнергии для космического корабля многоразового использования была мотивацией для эксперимента с привязанным спутником. Было предсказано, что ЭДС 5 кВ будет индуцироваться в 20-километровом тросе при движении с орбитальной скоростью в магнитном поле Земли. Цепь замыкается обратным трактом через неподвижную ионосферу.

Металлический стержень, вращающийся в магнитном поле На части (a) (Рисунок) показан металлический стержень OS , который вращается в горизонтальной плоскости вокруг точки O .Стержень скользит по проволоке, образуя дугу окружности PST радиусом r . Система находится в постоянном магнитном поле, направленном за пределы страницы. (a) Если вы вращаете стержень с постоянной угловой скоростью, каков ток I в замкнутом контуре OPSO ? Предположим, что резистор R обеспечивает все сопротивление в замкнутом контуре. (b) Рассчитайте работу за единицу времени, которую вы делаете при вращении стержня, и покажите, что она равна мощности, рассеиваемой в резисторе.

Стратегия Магнитный поток – это магнитное поле, умноженное на площадь четверти круга. При нахождении ЭДС по закону Фарадея все переменные постоянны во времени, но для расчета работы в единицу времени мы знаем, что это связано с крутящим моментом. умножить на угловую скорость. Крутящий момент рассчитывается исходя из силы, действующей на стержень, и ее интегрирования по длине стержня.

Решение

  1. Исходя из геометрии, площадь контура OPSO равна Следовательно, магнитный поток через контур равен


    Дифференцируя по времени и использованию, мы имеем


    При делении на сопротивление контура R получается величина индуцированного тока


    По мере увеличения увеличивается и поток через контур. Чтобы противодействовать этому увеличению, магнитное поле из-за индуцированного тока должно быть направлено на страницу в области, заключенной в контур.Следовательно, как показано в части (b) (Рисунок), ток циркулирует по часовой стрелке.

  2. Вы вращаете стержень, прилагая к нему крутящий момент. Поскольку стержень вращается с постоянной угловой скоростью, этот крутящий момент равен и противоположен крутящему моменту, приложенному к току в стержне исходным магнитным полем. Магнитная сила на бесконечно малом сегменте длиной dx , показанном в части (c) (Рисунок), такова, что магнитный момент на этом сегменте равен
    .


    Чистый магнитный крутящий момент на стержне будет


    Крутящий момент, который вы прилагаете к стержню, равен и противоположен ему, а работа, которую вы выполняете, когда стержень поворачивается на угол, равна Следовательно, работа на единицу времени, которую вы выполняете на стержне, составляет


    , где мы заменили I .Мощность, рассеиваемая в резисторе, может быть записана как


    Следовательно, мы видим, что


    Следовательно, мощность, рассеиваемая в резисторе, равна работе в единицу времени, совершаемой при вращении стержня.

Значение. Альтернативный способ взглянуть на индуцированную ЭДС из закона Фарадея – интегрировать в пространстве, а не во времени. Решение, однако, будет таким же. Двигательная ЭДС

Скорость может быть записана как угловая скорость, умноженная на радиус, а дифференциальная длина – как dr .Следовательно,

, это то же самое решение, что и раньше.

Проверьте свое понимание Стержень длиной 10 см движется со скоростью 10 м / с перпендикулярно через магнитное поле напряжением 1,5 Тл. Какая разница потенциалов между концами стержня?

Сводка

  • Связь между наведенной ЭДС в проводе, движущемся с постоянной скоростью v через магнитное поле B , задается формулой
  • Индуцированная ЭДС по закону Фарадея создается ЭДС движения, которая противодействует изменению потока.

Концептуальные вопросы

Барный магнит падает под действием силы тяжести вдоль оси длинной медной трубки. Если сопротивление воздуха незначительно, появится ли сила, препятствующая спуску магнита? Если да, достигнет ли магнит предельной скорости?

Вокруг географического Северного полюса (или южного магнитного полюса) магнитное поле Земли почти вертикальное. Если в этой области самолет летит на север, какая сторона крыла заряжена положительно, а какая отрицательно?

Положительные заряды на крыльях будут к западу или слева от пилота, а отрицательные заряды будут тянуться к востоку или справа от пилота.Таким образом, кончики левых крыльев будут положительными, а кончики правых – отрицательными.

Проволочная петля движется поступательно (без вращения) в однородном магнитном поле. В петле наведена ЭДС?

Глоссарий

ЭДС двигательного действия
напряжение, создаваемое движением проводника в магнитном поле

Электромагнитная индукция

Электромагнитная индукция

ВСЕ ТАБЛИЦЫ НА ОДНОЙ СТРАНИЦЕ ДЛЯ ЛЕГКОЙ ПЕЧАТИ

КОНКРЕТНЫЕ ЗАДАЧИ

Чтобы понять закон индукции Фарадея, закон Ленца и правило правой руки с помощью простых экспериментов.

ОБОРУДОВАНИЕ

Гальванометр, постоянный стержневой магнит, компас, две катушки соленоида (грубая и мелкая), железный стержень, алюминиевый стержень и аккумулятор.

ИСТОРИЯ

Провод, по которому проходит электрический ток, создает магнитное поле вокруг провод. Если провод свернут в спираль, называемую соленоидом, тогда магнитное поле напоминает магнитное поле стержневого магнита. Один из концов соленоида будет вести себя как северный (ищущий) полюс. стержневого магнита, а другой конец будет вести себя как южный полюс.Полярность зависит от направления тока и задается

ПРАВИЛО ПРАВОЙ РУКИ – Если взять катушку правой рукой так, чтобы пальцы указывают в направлении тока, большой палец указывает ближе к концу, который ведет себя как северный полюс. Противоположный конец ведет себя как Южный полюс.

EMF – Electro-Motive Force – старое название создаваемого напряжения не батареей, а изменяющимся магнитным полем. ЭДС может подтолкнуть электронов в проводнике, так как напряжение батареи может подтолкнуть электроны через провод.

ЭЛЕКТРОМАГНИТНАЯ ИНДУКЦИЯ – Всякий раз, когда проводник лежит в магнитное поле, в нем есть наведенная ЭДС. Если проводящий путь замкнутая цепь, ток будет течь. Величина тока зависит от от ЭДС (напряжения) и сопротивления цепи (помните закон Ома). В направление тока совпадает с направлением наведенной ЭДС.

ЗАКОН ИНДУКЦИИ ФАРАДЕЯ для ЭДС, индуцированной в проводящей петле, имеет вид

ЭДС = – N d / dt то есть величина наведенной ЭДС зависит от количества витков провода N в петле и от скорости изменения магнитного поток, который является продуктом магнитное поле В, и площадь петли.Поток может измениться так как
  1. магнитное поле меняется – B увеличивается или уменьшается
  2. Площадь петли меняется – меняется форма
  3. ориентация контура относительно направления магнитного поля меняется.
Направление наведенной ЭДС задается законом Ленца.

ЗАКОН ЛЕНЦА – всякий раз, когда ток течет в результате наведенной ЭДС, его направление таково, чтобы установить магнитный поток, противодействующий изменение условий возникновения наведенной ЭДС.Индуцированный ток стремится поддерживать постоянный поток через контур.

ГАЛЬВАНОМЕТР – очень чувствительный датчик тока, который может считывать ток, проходящий через него в любом направлении. Игла кончит по направлению к входящему току. (Представьте себе флюгер, указывающий на ветер.)

FERROMAGNETIC – описывает вещество, обладающее сильным магнитным эффекты. Образец будет привлечен либо к северному, либо к южному полюсу. магнита.

ПРОЦЕДУРА

  1. Обратите внимание на полярность стержневого магнита. Наблюдайте за эффектом планки магнит на компасе. Стрелка компаса указывает на или от северного полюса стержневого магнита?
  2. Предусмотрены две катушки соленоида: первичная с несколькими витками грубая проволока, а вторичная – с большим количеством витков тонкой проволоки.
  3. Обратите внимание на направление намотки первичной обмотки. Раньше на самом деле попробовав, спрогнозируй, используя правило правой руки, как должна быть батарея подключен к первичной обмотке для создания северного полюса на узком конце.Проверять ваше предсказание с компасом. Поменяйте ориентацию батареи и убедитесь, что южный полюс создается на узком конце первичной обмотки.
  4. Подсоедините вторичную катушку к гальванометру.
  5. Обратите внимание на направление намотки вторичной обмотки. Раньше на самом деле проводя эти эксперименты, с помощью закона Ленца вывести направление, в котором стрелка гальванометра отклонится.
  6. Результаты экспериментов в этой лаборатории являются качественными, а не количественными.Вы запишите направление тока через гальванометр. (вправо или влево) и относительная величина отклонения гальванометра индикатор (большой или маленький).
  7. Следующие инструкции кратко изложены в таблицы, в которых используются следующие сокращения
    • НП – северный полюс
    • SP – южный полюс
    • M – магнит
    • П – первичный соленоид
    • S – вторичный соленоид

  8. Быстро переместите северный полюс магнита во вторичную обмотку и обратите внимание направление и размер прогиба.Быстро уберите северный полюс от вторичной обмотки и отметьте направление и величину отклонения.
  9. Медленно переместите северный полюс магнита во вторичную обмотку и обратите внимание направление и размер прогиба. Медленно удалите северный полюс от вторичной обмотки и отметьте направление и величину отклонения.
  10. Быстро переместите южный полюс магнита во вторичную обмотку и обратите внимание направление и размер прогиба. Быстро уберите южный полюс от вторичной обмотки и отметьте направление и величину отклонения.
  11. Медленно переместите южный полюс магнита во вторичную обмотку и обратите внимание направление и размер прогиба. Медленно снимите южный полюс от вторичной обмотки и отметьте направление и величину отклонения.
  12. Обратите внимание, показывает ли гальванометр отклонение, когда северный полюс Магнит находится внутри вторичной обмотки, но не движется относительно нее.
  13. Нарисуйте большую четкую схему для обозначения ваших экспериментов. Включите полярность и направление движения магнита, в каком смысле вторичная обмотка намотана, и направление индуцированного тока в вторичный.Одной диаграммы может быть достаточно, если она достаточно общая.
  14. Объясните свои результаты, используя закон Ленца. Обобщайте как можно больше чтобы не перечислять каждый отдельный случай отдельно.

  15. Поместите первичную обмотку полностью внутрь вторичной. Используйте закон Ленца, чтобы предсказать результаты следующих экспериментов. Подключите аккумулятор к первичный, так что северный полюс создается на узком конце начальный. Обратите внимание на направление и размер отклонения. Отключите аккумулятор и отметьте направление и размер отклонения.
  16. Подключите батарею к первичному так, чтобы южный полюс образовался на узкий конец первичной обмотки. Обратите внимание на направление и размер прогиб. Отсоедините аккумулятор и обратите внимание на направление и размер прогиб.
  17. Обратите внимание на отклонение, когда батарея подключена к первичной и цепь не замыкается или не размыкается, то есть когда ток через первичный постоянный.
  18. Соответствуют ли ваши экспериментальные результаты с использованием первичного результаты использования стержневого магнита? Объяснять.
  19. Поместите алюминиевый стержень (более легкий из двух) внутрь первичного который все еще находится внутри вторичного. Алюминий – хороший проводник электричества, но это не ферромагнетик. Подключите аккумулятор к первичный, так что северный полюс создается на узком конце начальный. Обратите внимание на направление и размер отклонения. Отключите аккумулятор и отметьте направление и размер отклонения.
  20. Подключите батарею к первичному так, чтобы южный полюс образовался на узкий конец первичной обмотки.Обратите внимание на направление и размер прогиб. Отсоедините аккумулятор и обратите внимание на направление и размер прогиб.
  21. Есть ли экспериментально измеряемая разница между заполненными воздухом первичный и первичный заполненный алюминием?
  22. Поместите железный стержень (более тяжелый из двух) внутрь первичной обмотки. который все еще находится внутри вторичного. Железо – посредственный проводник электричества, и это ферромагнетик. Подключите аккумулятор к первичный, так что северный полюс создается на узком конце начальный.Обратите внимание на направление и размер отклонения. Отключите аккумулятор и отметьте направление и размер отклонения.
  23. Подключите батарею к первичному так, чтобы южный полюс образовался на узкий конец первичной обмотки. Обратите внимание на направление и размер прогиб. Отсоедините аккумулятор и обратите внимание на направление и размер прогиб.
  24. Есть ли экспериментально измеряемая разница между заполненными воздухом первичная и железная первичная?
  25. Найдите в Справочнике по химии и физике CRC магнитный восприимчивость алюминия и любых соединений железа.Как вы думаете, что представляет собой это число?
  26. Нарисуйте большую четкую схему для обозначения ваших экспериментов. Включите направление намотки первичной обмотки, направление приложения ток через первичную обмотку, в том смысле, в котором вторичная обмотка рана и направление индуцированного тока во вторичной обмотке. А может быть достаточно одной диаграммы, если она достаточно общая.
  27. Объясните свои результаты, используя закон Ленца.

  28. Снимите первичную обмотку с неповрежденным железным сердечником из вторичной обмотки и поместите два соленоида на стол так, чтобы первичный клапан был перпендикулярен середина средней школы.Подключите аккумулятор к первичной цепи так, чтобы северный полюс создается на узком конце первичной обмотки. Обратите внимание на направление и размер прогиба. Отсоедините аккумулятор и обратите внимание на направление и размер прогиба.
  29. Объясните свои результаты, используя закон Ленца.
Не забывайте свои два случайных и два систематических источника ошибок.
Назад к руководству по электричеству и магнетизму

13.4: ЭДС движения – Physics LibreTexts

Цели обучения

К концу этого раздела вы сможете:

  • Определить величину наведенной ЭДС в проводе, движущемся с постоянной скоростью через магнитное поле.
  • Обсудите примеры, в которых используется ЭДС движения, например, рельсовая пушка и привязанный спутник.

Магнитный поток зависит от трех факторов: силы магнитного поля, площади, через которую проходят силовые линии, и ориентации поля с площадью поверхности.Если какая-либо из этих величин изменяется, происходит соответствующее изменение магнитного потока. До сих пор мы рассматривали только изменения потока из-за изменяющегося поля. Теперь мы рассмотрим другую возможность: изменение области, через которую проходят силовые линии, включая изменение ориентации области.

Два примера этого типа изменения потока представлены на рисунке \ (\ PageIndex {1} \). В части (а) поток через прямоугольную петлю увеличивается по мере того, как она движется в магнитное поле, а в части (b) поток через вращающуюся катушку изменяется в зависимости от угла \ (\ theta \).

Рисунок \ (\ PageIndex {1} \): (а) Магнитный поток изменяется, когда петля движется в магнитное поле; (б) магнитный поток изменяется при вращении петли в магнитном поле.

Интересно отметить, что то, что мы воспринимаем как причину определенного изменения потока, на самом деле зависит от выбранной нами системы отсчета. Например, если вы находитесь в состоянии покоя относительно движущихся катушек на рисунке \ (\ PageIndex {1b} \), вы увидите, что поток меняется из-за изменения магнитного поля – в части (а) поле перемещается слева направо. прямо в вашей системе отсчета, и в части (b), поле вращается.Часто можно описать изменение магнитного потока через катушку, которая движется в одной конкретной системе отсчета, в терминах изменяющегося магнитного поля во второй системе отсчета, где катушка неподвижна. Однако вопросы системы отсчета, связанные с магнитным потоком, выходят за рамки этого учебника. Мы избежим таких сложностей, всегда работая в кадре в состоянии покоя относительно лаборатории и объясняя вариации потока как следствие либо изменяющегося поля, либо изменяющейся области.

Теперь давайте посмотрим на проводящий стержень, включенный в цепь, изменяющую магнитный поток.Площадь, ограниченная схемой MNOP на рисунке \ (\ PageIndex {2} \), составляет лк и перпендикулярна магнитному полю, поэтому мы можем упростить интегрирование \ (\ Phi _ {\ mathrm {m}} = \ int_ {S} \ overrightarrow {\ mathbf {B}} \ cdot \ hat {\ mathbf {n}} d A \) в умножение магнитного поля и площади. Следовательно, магнитный поток через открытую поверхность составляет

.

\ [\ Phi_m = Blx. \]

Поскольку B и l постоянны, а скорость стержня равна \ (v = dx / dt \), теперь мы можем переформулировать закон Фарадея, уравнение 13.2.2, для величины ЭДС в движущемся проводящем стержне как

\ [\ epsilon = \ frac {d \ Phi_m} {dt} = Bl \ frac {dx} {dt} = Blv. \]

Ток, наведенный в цепи, равен ЭДС, деленной на сопротивление, или

\ [I = \ frac {Blv} {R}. \]

Кроме того, направление индуцированной ЭДС удовлетворяет закону Ленца, что вы можете проверить, посмотрев на рисунок.

Этот расчет ЭДС, вызванной движением, не ограничивается перемещением стержня по проводящим рельсам.Используя \ (\ vec {F} = q \ vec {v} \ times \ vec {B} \) в качестве отправной точки, можно показать, что \ (\ epsilon = – d \ Phi_m / dt \) выполняется для любого изменение магнитного потока, вызванное движением проводника. Мы видели в законе Фарадея, что ЭДС, индуцированная изменяющимся во времени магнитным полем, подчиняется той же зависимости, которая является законом Фарадея. Таким образом, закон Фарадея справедлив для всех изменений магнитного потока , независимо от того, вызваны ли они изменяющимся магнитным полем, движением или их комбинацией.

Рисунок \ (\ PageIndex {2} \): проводящий стержень смещается вправо с постоянной скоростью.2} {R}. \]

В соответствии с принципом сохранения энергии производимая и рассеиваемая мощности равны.

Этот принцип можно увидеть в работе рельсового пистолета. Рельсовая пушка представляет собой пусковую установку электромагнитных снарядов, в которой используется устройство, подобное изображенному на рисунке \ (\ PageIndex {2} \), и схематично показано на рисунке \ (\ PageIndex {3} \). Проводящий стержень заменяется выстрелом или оружием. До сих пор мы слышали только о том, как движение вызывает ЭДС.В рельсовой пушке оптимальное отключение / снижение магнитного поля уменьшает поток между рельсами, вызывая протекание тока в стержне (якорь), удерживающем снаряд. Этот ток через якорь испытывает магнитную силу и продвигается вперед. Однако рельсовые пушки не используются широко в вооруженных силах из-за высокой стоимости производства и больших токов: для выработки энергии, достаточной для того, чтобы рельсовая пушка была эффективным оружием, требуется около миллиона ампер.

Рисунок \ (\ PageIndex {3} \): Ток, протекающий по двум рельсам, движет токопроводящий снаряд вперед благодаря создаваемой магнитной силе.

Мы можем вычислить ЭДС, индуцированную движением, с помощью закона Фарадея , даже когда фактически замкнутый контур отсутствует . Мы просто представляем замкнутую область, граница которой включает движущийся проводник, вычисляем \ (\ Phi_m \), а затем находим ЭДС по закону Фарадея. Например, мы можем позволить движущемуся стержню на рисунке \ (\ PageIndex {4} \) находиться на одной стороне воображаемой прямоугольной области, представленной пунктирными линиями. Площадь прямоугольника составляет лк , поэтому магнитный поток через него равен \ (\ Phi_m = Blx \).Дифференцируя это уравнение, получаем

\ [\ frac {d \ Phi_m} {dt} = Bl \ frac {dx} {dt} = Blv, \]

, что соответствует разности потенциалов между концами стержня, которую мы определили ранее.

Рисунок \ (\ PageIndex {4} \): с показанным воображаемым прямоугольником мы можем использовать закон Фарадея для вычисления наведенной ЭДС в движущемся стержне.

ЭДС движения в слабом магнитном поле Земли обычно не очень велики, иначе мы могли бы заметить напряжение на металлических стержнях, таких как отвертка, во время обычных движений.{-5} T) (1.0 \, m) (3.0 \, m / s) = 150 \, \ mu V. \]

Это небольшое значение согласуется с опытом. Однако есть впечатляющее исключение. В 1992 и 1996 годах с космическим шаттлом были предприняты попытки создать большие двигательные ЭДС. Привязанный спутник должен был быть выпущен на 20-километровом проводе, как показано на рисунке \ (\ PageIndex {5} \), для создания ЭДС 5 кВ за счет движения с орбитальной скоростью через поле Земли. Эту ЭДС можно было бы использовать для преобразования некоторой кинетической и потенциальной энергии шаттла в электрическую, если бы можно было создать полную схему.Чтобы замкнуть цепь, неподвижная ионосфера должна была обеспечить обратный путь, по которому мог течь ток. (Ионосфера – это разреженная и частично ионизированная атмосфера на орбитальных высотах. Она проводит из-за ионизации. Ионосфера выполняет ту же функцию, что и стационарные рельсы и соединительный резистор на рисунке \ (\ PageIndex {3} \), без которых не было бы Это не полная цепь.) Затяните ток в кабеле из-за магнитной силы \ (F = IlBsin \, \ theta \) выполняет работу, которая уменьшает кинетическую и потенциальную энергию шаттла и позволяет преобразовать ее в электрическую. энергия.Оба теста не увенчались успехом. В первом случае кабель завис, и его можно было протянуть только на пару сотен метров; во втором трос оборвался при почти полном растяжении. Пример \ (\ PageIndex {1} \) указывает на принципиальную осуществимость.

Рисунок \ (\ PageIndex {5} \): ЭДС движения как преобразование электроэнергии для космического челнока была мотивацией для эксперимента с привязанным спутником. Было предсказано, что ЭДС 5 кВ будет индуцироваться в 20-километровом тросе при движении с орбитальной скоростью в магнитном поле Земли.o \) и поэтому \ (sin \, \ theta = 1 \).

Пример \ (\ PageIndex {2} \): металлический стержень, вращающийся в магнитном поле

В части (а) рисунка \ (\ PageIndex {6} \) показан металлический стержень OS , который вращается в горизонтальной плоскости вокруг точки O . Стержень скользит по проволоке, которая образует дугу окружности PST радиусом r . Система находится в постоянном магнитном поле \ (\ vec {B} \), которое направлено за пределы страницы. (а) Если вы вращаете стержень с постоянной угловой скоростью \ (\ omega \), каков ток I в замкнутом контуре OPSO ? Предположим, что резистор R обеспечивает все сопротивление в замкнутом контуре.2 \ omega} {2R}. \] По мере увеличения \ (\ theta \) увеличивается и поток через петлю из-за \ (\ vec {B} \). Чтобы противодействовать этому увеличению, магнитное поле из-за индуцированного тока должно быть направлено на страницу в области, ограниченной петлей. Следовательно, как показано в части (b) рисунка \ (\ PageIndex {6} \), ток циркулирует по часовой стрелке.

  • Вы вращаете стержень, прилагая к нему крутящий момент. Поскольку стержень вращается с постоянной угловой скоростью, этот крутящий момент равен и противоположен крутящему моменту, приложенному к току в стержне исходным магнитным полем.2} {4R}. \] Следовательно, мы видим, что \ [P = \ frac {dW} {dt}. \] Следовательно, мощность, рассеиваемая в резисторе, равна работе, совершаемой в единицу времени при вращении стержня.
  • Значение

    Альтернативный способ взглянуть на индуцированную ЭДС из закона Фарадея – интегрировать в пространстве, а не во времени. Решение, однако, будет таким же. Двигательная ЭДС

    \ [| \ epsilon | = \ int Bvdl. \]

    Скорость может быть записана как угловая скорость, умноженная на радиус, а дифференциальная длина – как dr .2, \], что является тем же решением, что и раньше.

    Пример \ (\ PageIndex {3} \): прямоугольная катушка, вращающаяся в магнитном поле

    Прямоугольная катушка площадью A, и N, витков помещена в однородное магнитное поле \ (\ vec {B} = B \ hat {j} \), как показано на рисунке \ (\ PageIndex {7} \ ). Катушка вращается вокруг оси z через ее центр с постоянной угловой скоростью \ (\ omega \). Получите выражение для наведенной ЭДС в катушке.

    Рисунок \ (\ PageIndex {7} \): прямоугольная катушка, вращающаяся в однородном магнитном поле.

    Стратегия

    Согласно диаграмме, угол между перпендикуляром к поверхности ((\ hat {n} \)) и магнитным полем \ ((\ vec {B}) \) равен \ (\ theta \). Скалярное произведение \ (B \ cdot \ hat {n} \) упрощается только до компонента \ (cos \, \ theta \) магнитного поля, а именно там, где магнитное поле проецируется на вектор единичной площади \ (\ hat {n} \). Величина магнитного поля и площадь контура фиксируются во времени, что позволяет быстро упростить интеграцию.Индуцированная ЭДС записывается с использованием закона Фарадея.

    Решение

    Когда катушка находится в таком положении, что ее нормальный вектор \ (\ hat {n} \) составляет угол \ (\ theta \) с магнитным полем \ (\ vec {B} \), магнитный поток через одиночный виток катушки

    \ [\ Phi_m = \ int_S \ vec {B} \ cdot \ hat {n} dA = BA \, cos \, \ theta. \]

    Согласно закону Фарадея, ЭДС, индуцированная в катушке, равна

    \ [\ epsilon = – N \ frac {d \ Phi_m} {dt} = NB A \, sin \, \ theta \ frac {d \ theta} {dt}.\]

    Постоянная угловая скорость равна \ (\ omega = d \ theta / dt \). Угол \ (\ theta \) представляет собой изменение угловой скорости во времени или \ (\ omega t \). Это меняет функцию на время-пространство, а не на \ (\ theta \). Следовательно, индуцированная ЭДС синусоидально изменяется со временем в соответствии с

    .

    \ [\ epsilon = \ epsilon_0 \, sin \, \ omega t, \], где \ (\ epsilon_0 = NBA \ omega \).

    Значение

    Если бы напряженность магнитного поля или площадь контура также менялись со временем, эти переменные нельзя было бы извлечь из производной по времени в простое решение, как показано.Этот пример является основой для электрического генератора, поскольку мы подробно рассмотрим его в разделе «Применение закона Ньютона».

    Упражнение \ (\ PageIndex {1} \)

    Ниже показан стержень длиной l , который вращается против часовой стрелки вокруг оси на o крутящим моментом, обусловленным \ (m \ vec {g} \). 2 \ omega / 2 \), с O при более высоком потенциале, чем S

    Упражнение \ (\ PageIndex {2} \)

    Стержень длиной 10 см движется со скоростью 10 м / с перпендикулярно через 1.Магнитное поле 5 Тл. Какая разница потенциалов между концами стержня?

    Ответ

    1,5 В

    Авторы и авторство

    Сэмюэл Дж. Линг (Государственный университет Трумэна), Джефф Санни (Университет Лойола Мэримаунт) и Билл Мобс со многими авторами. Эта работа лицензирована OpenStax University Physics в соответствии с лицензией Creative Commons Attribution License (4.0).

    Страница не найдена | MIT

    Перейти к содержанию ↓
    • Образование
    • Исследовать
    • Инновации
    • Прием + помощь
    • Студенческая жизнь
    • Новости
    • Выпускников
    • О MIT
    • Подробнее ↓
      • Прием + помощь
      • Студенческая жизнь
      • Новости
      • Выпускников
      • О MIT
    Меню ↓ Поиск Меню Ой, похоже, мы не смогли найти то, что вы искали!
    Попробуйте поискать что-нибудь еще! Что вы ищете? Увидеть больше результатов

    Предложения или отзывы?

    Закон электромагнитной индукции Фарадея | Электромагнетизм

    10.3 Закон электромагнитной индукции Фарадея (ESBPY)

    Ток, индуцированный изменяющимся магнитным полем (ESBPZ)

    Хотя удивительное открытие электромагнетизма Эрстедом проложило путь для более практических применений электричества, именно Майкл Фарадей дал нам ключ к практическому производству электричества: электромагнитная индукция .

    Фарадей обнаружил, что когда он перемещал магнит рядом с проводом, на нем генерировалось напряжение. Если магнит удерживался в неподвижном состоянии, напряжение не генерировалось, оно существовало только во время движения магнита.Мы называем это напряжение индуцированной ЭДС (\ (\ mathcal {E} \)).

    Цепной контур, подключенный к чувствительному амперметру, будет регистрировать ток, если он настроен, как показано на этом рисунке, и магнит перемещается вверх и вниз:

    Магнитный поток

    Прежде чем мы перейдем к определению закона электромагнитной индукции Фарадея и примерам, нам сначала нужно потратить некоторое время на изучение магнитного потока. Для петли площадью \ (A \) в присутствии однородного магнитного поля \ (\ vec {B} \) магнитный поток (\ (φ \)) определяется как: \ [\ phi = BA \ cos \ theta \] Где: \ begin {align *} \ theta & = \ text {угол между магнитным полем B и нормалью к петле в области A} \\ A & = \ text {область петли} \\ B & = \ text {магнитное поле} \ end {align *}

    Отель S.I. единица магнитного потока – Вебер (Вб).

    Вы можете спросить себя, почему включен угол \ (\ theta \). Поток зависит от магнитного поля, проходящего через поверхность. Мы знаем, что поле, параллельное поверхности, не может вызвать ток, потому что оно не проходит через поверхность. Если магнитное поле не перпендикулярно поверхности, то есть компонент, который перпендикулярен, и компонент, который параллелен поверхности. Параллельная составляющая не может вносить вклад в поток, только вертикальная составляющая может.

    На этой диаграмме мы показываем, что магнитное поле под углом, отличным от перпендикулярного, может быть разбито на составляющие. Компонент, перпендикулярный поверхности, имеет величину \ (B \ cos (\ theta) \), где \ (\ theta \) – угол между нормалью и магнитным полем.

    Закон электромагнитной индукции Фарадея

    ЭДС \ (\ mathcal {E} \), создаваемая вокруг контура проводника, пропорциональна скорости изменения магнитного потока φ через площадь A контура.Математически это можно выразить как:

    \ [\ mathcal {E} = -N \ frac {\ Delta \ phi} {\ Delta t} \]

    где \ (\ phi = B · A \), а B – напряженность магнитного поля. \ (N \) – количество контуров схемы. Магнитное поле измеряется в теслах (Тл). Знак минус указывает направление и то, что наведенная ЭДС имеет тенденцию противодействовать изменению магнитного потока. Знак минус можно не учитывать при вычислении звездных величин.

    Закон Фарадея связывает наведенную ЭДС со скоростью изменения магнитного потока, который является произведением магнитного поля и площади поперечного сечения, через которое проходят силовые линии.

    Это не площадь самого провода, а площадь, которую он ограничивает. Это означает, что если вы согнете проволоку в круг, площадь, которую мы будем использовать при вычислении потока, будет площадью поверхности круга, а не проволоки.

    На этом рисунке, где магнит находится в той же плоскости, что и контур цепи, не будет тока, даже если магнит перемещается все ближе и дальше. Это связано с тем, что силовые линии магнитного поля не проходят через замкнутое пространство, а параллельны ему.Силовые линии магнитного поля должны проходить через область, ограниченную контуром цепи, чтобы возникла ЭДС.

    Направление наведенного тока (ESBQ2)

    Самая важная вещь, которую следует помнить, это то, что индуцированный ток противостоит происходящим изменениям.

    На первом рисунке (слева) контурная петля имеет южный полюс приближающегося магнита. Величина поля от магнита становится больше. Реакция наведенной ЭДС будет состоять в том, чтобы попытаться противодействовать усилению поля по направлению к полюсу.Поле является вектором, поэтому ток будет течь в таком направлении, что поля, возникающие из-за тока, имеют тенденцию нейтрализовать поля от магнита, сохраняя результирующее поле неизменным.

    Чтобы противостоять переходу от приближающегося южного полюса сверху, ток должен приводить к силовым линиям, удаляющимся от приближающегося полюса. Поэтому индуцированное магнитное поле должно иметь силовые линии, идущие вниз по внутренней стороне петли. Направление тока, указанное стрелками на контуре цепи, будет достигнуто.Проверьте это, используя Правило правой руки. Поместите большой палец правой руки в направлении одной из стрелок и обратите внимание на то, что поле закручивается вниз в область, ограниченную петлей.

    На второй диаграмме южный полюс удаляется. Это означает, что поле от магнита станет слабее. Отклик от индуцированного тока будет заключаться в создании магнитного поля, которое добавляется к существующему от магнитного поля, чтобы противостоять его уменьшению в силе.

    Другой способ представить ту же функцию – просто использовать полюса.Чтобы противостоять приближающемуся южному полюсу, индуцируемый ток создает поле, которое выглядит как еще один южный полюс со стороны приближающегося южного полюса. Подобно отталкиванию полюсов, вы можете представить себе, как течение создает южный полюс, чтобы отразить приближающийся южный полюс. На второй панели ток устанавливает северный полюс, чтобы привлечь южный полюс и остановить его движение.

    Мы также можем использовать вариант правила правой руки, помещая пальцы в направлении течения, чтобы большой палец указывал в направлении силовых линий (или северного полюса).

    Мы можем проверить все это на случаях, когда северный полюс перемещается ближе или дальше от цепи. В первом случае приближения северного полюса ток будет сопротивляться изменению, создавая поле в направлении, противоположном полю, исходящему от магнита, который становится сильнее. Используйте Правило правой руки, чтобы убедиться, что стрелки создают поле с линиями поля, которые изгибаются вверх в замкнутой области, нейтрализуя те, которые изгибаются вниз от северного полюса магнита.

    Подобно тому, как полюса отталкиваются, в качестве альтернативы проверьте, что если поместить пальцы правой руки в направлении течения, ваш большой палец будет указывать вверх, указывая на северный полюс.

    Для второго рисунка, где северный полюс удаляется, ситуация обратная.

    Направление индуцированного тока в соленоиде (ESBQ3)

    Подход к изучению направления тока в соленоиде аналогичен подходу, описанному выше. Единственная разница в том, что в соленоиде есть несколько витков проволоки, поэтому величина наведенной ЭДС будет другой.Поток будет рассчитываться с использованием площади поверхности соленоида, умноженной на количество петель.

    Помните: направления токов и связанных с ними магнитных полей можно найти, используя только Правило правой руки. Когда пальцы правой руки направлены в направлении магнитного поля, большой палец указывает в направлении тока. Когда большой палец направлен в направлении магнитного поля, пальцы указывают в направлении тока.

    Направление тока будет таким, чтобы препятствовать изменению. Мы бы использовали установку, как в этом скетче, для проведения теста:

    В случае, когда северный полюс направлен к соленоиду, ток будет течь так, чтобы северный полюс установился на конце соленоида, ближайшем к приближающемуся магниту, чтобы оттолкнуть его (проверьте, используя Правило правой руки):

    В случае, когда северный полюс движется от соленоида, ток будет течь так, что южный полюс будет установлен на конце соленоида, ближайшем к удаляющемуся магниту, чтобы притягивать его:

    В случае, когда южный полюс движется от соленоида, ток будет течь так, что северный полюс будет установлен на конце соленоида, ближайшем к удаляющемуся магниту, чтобы притягивать его:

    В случае, когда южный полюс направлен к соленоиду, ток будет течь так, что южный полюс будет установлен на конце соленоида, ближайшем к приближающемуся магниту, чтобы оттолкнуть его:

    Простой способ создать магнитное поле изменяющейся интенсивности – переместить постоянный магнит рядом с проволокой или катушкой с проволокой.Магнитное поле должно увеличиваться или уменьшаться по напряженности перпендикулярно к проводу (так, чтобы силовые линии магнитного поля «пересекали» проводник), иначе не будет индуцироваться напряжение.

    Индуцированный ток создает магнитное поле. Индуцированное магнитное поле имеет направление, которое стремится нейтрализовать изменение магнитного поля в петле из проволоки. Итак, вы можете использовать Правило правой руки, чтобы найти направление индуцированного тока, помня, что индуцированное магнитное поле противоположно направлению изменения магнитного поля.

    Индукция

    Электромагнитная индукция находит практическое применение в конструкции электрических генераторов, которые используют механическую энергию для перемещения магнитного поля мимо катушек с проволокой для генерации напряжения. Однако это далеко не единственное практическое применение этого принципа.

    Если мы вспомним, магнитное поле, создаваемое проводом с током, всегда перпендикулярно проводу, и что сила потока этого магнитного поля зависит от величины тока, который проходит через него.Таким образом, мы можем видеть, что провод может создавать напряжение на своей собственной длине , если ток изменяется. Этот эффект называется самоиндукцией . Самоиндукция – это когда изменяющееся магнитное поле создается изменением тока через провод, вызывая напряжение по длине того же провода.

    Если магнитный поток усиливается путем сгибания проволоки в форме катушки и / или наматывания этой катушки на материал с высокой проницаемостью, этот эффект самоиндуцированного напряжения будет более интенсивным.Устройство, созданное для использования этого эффекта, называется дросселем .

    Помните, что индуцированный ток создает магнитное поле, которое противодействует изменению магнитного потока. Это известно как закон Ленца.

    Рабочий пример 1: закон Фарадея

    Рассмотрим плоскую квадратную катушку с 5 витками. Катушка находится \ (\ text {0,50} \) \ (\ text {m} \) с каждой стороны и имеет магнитное поле \ (\ text {0,5} \) \ (\ text {T} \) проходя через него. Плоскость катушки перпендикулярна магнитному полю: поле направлено за пределы страницы.Используйте закон Фарадея для вычисления наведенной ЭДС, если магнитное поле увеличивается равномерно от \ (\ text {0,5} \) \ (\ text {T} \) до \ (\ text {1} \) \ (\ текст {T} \) в \ (\ text {10} \) \ (\ text {s} \). Определите направление индуцированного тока.

    Определите, что требуется

    Мы обязаны использовать Закон Фарадея для расчета наведенной ЭДС.

    Запишите закон Фарадея

    \ [\ mathcal {E} = – N \ frac {\ Delta \ phi} {\ Delta t} \] Мы знаем, что магнитное поле расположено под прямым углом к ​​поверхности и поэтому выровнено с нормалью.Это означает, что нам не нужно беспокоиться об угле, который поле образует с нормалью и \ (\ phi = BA \). Начальное или начальное магнитное поле, \ (B_i \), задается как конечная величина поля, \ (B_f \). Мы хотим определить величину ЭДС, чтобы можно было игнорировать знак минус.

    Площадь \ (A \) – это площадь квадратной катушки. 2 (\ text {1} – \ text {0,50})} {\ text {10}} \\ & = \ текст {0,0625} \ текст {V} \ end {выровнять *}

    Индуцированный ток направлен против часовой стрелки, если смотреть со стороны нарастающего магнитного поля.

    Рабочий пример 2: закон Фарадея

    Рассмотрим соленоид из 9 витков с неизвестным радиусом \ (r \). На соленоид действует магнитное поле \ (\ text {0,12} \) \ (\ text {T} \). Ось соленоида параллельна магнитному полю. Когда поле равномерно переключается на \ (\ text {12} \) \ (\ text {T} \) в течение 2 минут, ЭДС величиной \ (- \ text {0,3} \) \ (\ text {V} \) индуцируется. Определите радиус соленоида.

    Определите, что требуется

    Требуется определить радиус соленоида.Мы знаем, что связь между наведенной ЭДС и полем регулируется законом Фарадея, который включает геометрию соленоида. Мы можем использовать это соотношение, чтобы найти радиус.

    Запишите закон Фарадея

    \ [\ mathcal {E} = – N \ frac {\ Delta \ phi} {\ Delta t} \] Мы знаем, что магнитное поле расположено под прямым углом к ​​поверхности и поэтому выровнено с нормалью. Это означает, что нам не нужно беспокоиться об угле, который поле образует с нормалью и \ (\ phi = BA \).{- \ text {2}} \) \ (\ text {m} \). Соленоид подвергается воздействию переменного магнитного поля, которое равномерно изменяется от \ (\ text {0,4} \) \ (\ text {T} \) до \ (\ text {3,4} \) \ (\ text { T} \) в интервале \ (\ text {27} \) \ (\ text {s} \). Ось соленоида составляет угол \ (\ text {35} \) \ (\ text {°} \) к магнитному полю. Найдите наведенную ЭДС.

    Определите, что требуется

    Мы обязаны использовать Закон Фарадея для расчета наведенной ЭДС.

    Запишите закон Фарадея

    \ [\ mathcal {E} = – N \ frac {\ Delta \ phi} {\ Delta t} \] Мы знаем, что магнитное поле расположено под углом к ​​нормали к поверхности.{- \ text {3}} \ text {V} \ end {выровнять *}

    Индуцированный ток направлен против часовой стрелки, если смотреть со стороны нарастающего магнитного поля.

    Реальные приложения

    Следующие устройства используют в своей работе закон Фарадея.

    • Плиты индукционные

    • магнитофонов

    • металлоискатели

    • трансформаторы

    Реальные применения закона Фарадея

    Выберите одно из следующих устройств и поищите в Интернете или библиотеке, как работает ваше устройство.В объяснении вам нужно будет сослаться на закон Фарадея.

    • Плиты индукционные

    • магнитофонов

    • металлоискатели

    • трансформаторы

    Вы справитесь! Позвольте нам помочь вам учиться с умом для достижения ваших целей. Siyavula Practice направит вас в удобном для вас темпе, когда вы задаете вопросы в Интернете.

    Зарегистрируйтесь, чтобы улучшить свои оценки

    Закон Фарадея

    Упражнение 10.2

    Изложите закон электромагнитной индукции Фарадея словами и запишите математическое соотношение.

    ЭДС \ (\ mathcal {E} \), создаваемая вокруг контура проводника, пропорциональна скорости изменения магнитного потока φ через площадь A контура. Математически это можно выразить как:

    \ [\ mathcal {E} = -N \ frac {\ Delta \ phi} {\ Delta t} \]

    где \ (\ phi = B · A \), а B – напряженность магнитного поля.\ (N \) – количество контуров схемы. Магнитное поле измеряется в теслах (Тл). Знак минус указывает направление и то, что наведенная ЭДС имеет тенденцию противодействовать изменению магнитного потока. Знак минус можно не учитывать при вычислении звездных величин.

    Опишите, что происходит, когда стержневой магнит вдавливается в соленоид, подключенный к амперметру, или вытягивается из него. Нарисуйте картинки, подтверждающие ваше описание.

    В случае, когда северный полюс направлен к соленоиду, ток будет течь так, чтобы северный полюс установился на конце соленоида, ближайшем к приближающемуся магниту, чтобы оттолкнуть его (проверьте, используя Правило правой руки):

    В случае, когда северный полюс движется от соленоида, ток будет течь так, что южный полюс будет установлен на конце соленоида, ближайшем к удаляющемуся магниту, чтобы притягивать его:

    В случае, когда южный полюс движется от соленоида, ток будет течь так, что северный полюс будет установлен на конце соленоида, ближайшем к удаляющемуся магниту, чтобы притягивать его:

    В случае, когда южный полюс направлен к соленоиду, ток будет течь так, что южный полюс будет установлен на конце соленоида, ближайшем к приближающемуся магниту, чтобы оттолкнуть его:

    Объясните, как магнитный поток может быть равен нулю, когда магнитное поле не равно нулю.

    Поток связан с магнитным полем:

    \ (\ phi = BA \ cos \ theta \)

    Если \ (\ cos \ theta \) равно 0, то магнитный поток будет равен 0, даже если есть магнитное поле. В этом случае магнитное поле параллельно поверхности и не проходит через нее.

    Используйте правило правой руки, чтобы определить направление индуцированного тока в соленоиде ниже.

    Южный полюс магнита приближается к соленоиду.Закон Ленца говорит нам, что ток будет течь, чтобы противодействовать изменению. Южный полюс на конце соленоида будет противодействовать приближающемуся южному полюсу. Ток будет циркулировать по странице в верхней части катушки, так что большой палец правой руки будет указывать влево.

    Рассмотрим круговую катушку из 5 витков с радиусом \ (\ text {1,73} \) \ (\ text {m} \). Катушка подвергается воздействию переменного магнитного поля, которое равномерно изменяется от \ (\ text {2,18} \) \ (\ text {T} \) до \ (\ text {12,7} \) \ (\ text { T} \) в интервале \ (\ text {3} \) \ (\ text {minutes} \). {2} & = \ текст {0,0479} \\ г & = \ текст {0,22} \ текст {м} \ end {выровнять *}

    Найдите изменение потока, если ЭДС равна \ (\ text {12} \) \ (\ text {V} \) за период \ (\ text {12} \) \ (\ text {s} \) .

    \ begin {align *} \ mathcal {E} & = N \ frac {\ Delta \ phi} {\ Delta t} \\ 12 & = 5 \ left (\ frac {\ Delta \ phi} {12} \ right) \\ \ Delta \ phi & = \ text {28,8} \ text {Wb} \ end {выровнять *}

    Если угол изменить на \ (\ text {45} \) \ (\ text {°} \), на какой временной интервал нужно изменить, чтобы наведенная ЭДС оставалась прежней?

    \ begin {align *} \ mathcal {E} & = N \ frac {\ Delta \ phi} {\ Delta t} \\ & = N \ frac {\ phi_ {f} – \ phi_ {i}} {\ Delta t} \\ & = N \ frac {B_ {f} A \ cos \ theta – B_ {i} A \ cos \ theta} {\ Delta t} \\ & = \ cos \ theta \ times N \ frac {B_ {f} A – B_ {i} A} {\ Delta t} \ end {выровнять *}

    Все значения остаются неизменными между двумя описанными ситуациями, за исключением угла и времени.Мы можем приравнять уравнения для двух сценариев:

    \ begin {align *} \ mathcal {E} _1 & = \ mathcal {E} _2 \\ \ cos \ theta_1 \ times N \ frac {B_ {f} A – B_ {i} A} {\ Delta t_1} & = \ cos \ theta_2 \ times N \ frac {B_ {f} A – B_ {i} A } {\ Delta t_2} \\ \ cos \ theta_1 \ frac {1} {\ Delta t_1} & = \ cos \ theta_2 \ frac {1} {\ Delta t_2} \\ \ Delta t_2 & = \ frac {\ Delta t_1 \ cos \ theta_2} {\ cos \ theta_1} \\ \ Delta t_2 & = \ frac {(\ text {12} \ cos (\ text {45}} {\ cos (\ text {23})} \\ \ Delta t_2 & = \ text {9,22} \ text {s} \ end {выровнять *} .

    Добавить комментарий

    Ваш адрес email не будет опубликован. Обязательные поля помечены *